Anda di halaman 1dari 110

SOAL CBT BLOK 16

1. Tes kehamilan (hcg) dihasilkan oleh sel apa?


a. Sinsiotropoblast

b. Korpus luteum

2. Hormone yang mempertahankan kehamilan ?


a. Estrogen
b. Progesteron

c. Hcg

3. Wanita keluhan gatal gatal pada alat kelamin dan keputihan banyak,
apa pemeriksaan selanjutnya yang dilakukan?
a. Pap smear

b. Vaginal swab

4. Antihipertensi yang dapat menurunkan curah jantung dan resistensi


perifer?
a. Labetolol
b. Nifedipin

c. Hidroliazin

5. Ibu datang pasca persalinan dengan keluhan perdarahan. Pemeriksaan


fisik uterus terasa lunak. Dan pada inspeksi didapatkan jalan lahir ada
bekas jahitan. Kira-kira apa penyebab perdarahan?
a. Laserasi jalan lahir
b. Ruptura Uteri

c. Atonia Uteri

6. Yang mempertahankan ligamentum tetap relaksasi?


a. Estrogen
b. Progesteron

c. Oksitosin
d. Relaksin

7. Ibu hamil dengan suami perokok dan riwayat abortus sebelumnya. Itu
merupakan faktor resiko untuk :
a. Preeklampsia
b. Eklampsia

c. Lahir prematur

8. Ibu hamil, umur kehamilan 8-9 minggu. Keluhan nyeri dan perdarahan,
Darah cavum douglas kuldosintesis (+) . apa kira-kira diagnosanya ?
a. Missed abortion

b. Kehamilan
terganggu

ektopik

9. Otot pada perineum, apa otot yg ditunjuuuuk?


a. m.koksigeus
b. m.pubokoksigeus
c. m.iliokoksigeus

d. m.tranfersusu
profunda
1

perinalis

10.
Ibu, persalinan bukaan udah 6 cm, janin udah masuk bidang
hodge 3, ketuban belum pecah .. apa yang harus dilakukaan?
a. Observasi
b. Oksitosin drip

c. Vakum
d. Forcep

11.
Ibu hamil >20 minggu, riwayat hipertensi, keluhan perdarahan
dan nyeri. Diagnosa?
a. Solusio placenta (darah
kehitaman, nyeri, perut
tegang, darah dikit)

b. Plasenta
VT)
c. KET
d. Abortus

previa (boleh di

12.
Ibu hamil, lahirin di dukun sampe 2 jam, bayi ga keluar-keluar ..
ke rumah sakit, ternyata bayi udah masuk bidang hodge 4. Apa yang
harus dilakukan?
a. Focep (delee, kpala udh
didasar panggung, rotasi
interna
sempurna,

muskulus
levator
teregang)
b. Induksi
c. Vakum

ani

13.
Ibu punya bayi umur 8 bulan. Memakai kontrasepsi hormonal
yang progestin, ingin mengganti kontrasepsi yang lain. Dokter
menyarankan kontrasepsi oral kombinasi. Efek estrogen dalam
kontrasepsi tersebut?
a. Supresi LH
14.

b. Supresi FSH

Hiperemis gravidarum diikuti gejala?

a. Hipertensi
b. Diare
15.

c. Hipokalemiakarena
muntah

Kenapa hamil sering mikssiiii?

a. Penekanan uterus
b. Klo
awal
awal
progesteron

c. Efek estrogen
meningkat

tonus

16.
Datang keluhan perdarahan banyak saat menstruasi, sikluas haid
teratur. Riwayat menoragi. Keluhan konstipasi. Px fisik ditemukan
masa suprasimfisis kira-kira 20 cm. Kira-kira apa diagnosanyaa?
a. Mioma uteri
b. Kista ovarium

c. Ca cervix

17.
Ibu hamil 40 minggu, datang dengan keluhan uterus kontraksi
terus menerus. Didapatkan kontraksi irreguler. Dan pembukaan sudah
2

1 cm. Ibu ini mengeluhkan hal yang sama beberapa minggu yang lalu.
a.
Fase laten
kala 1
c. Fase deselerasi
Diagnosa
:
b. Fase aktif kala 1
d. Flase labour
18.

Ibu Perdarahan setelah melahirkan. Kemungkinan kekeurangan :

a. Zat besi
b. Magnesium

c. Vitamin K

19.
Yang mempengaruhi payudara menjadi tegang?
a. Estrogen, progesteron, somatotropin
20.
Soal bonus anatomi vaskuarisasi
21.
Ibu datang dengan keluhan nyeri kuadran kanan bawah, positif
darah di kavum douglas . yang harus dilakukan selanjutnya?
a. Laparotomi
b. Kuratase

c. Pemberian cairan

22.
Ibu hamil 34 minggu .. Pada saat ANC tekanan darah 150 mmHG.
Selama kehamilan rata-rata 100-120/70-90 mmHg . Pada saat
anamnesis disangkal adanya pengelihatan kabur, pusing. Pemeriksaan
lab proteinuri (-), trombosit 160.000, ALT : 22, AST :15 .. Diagnosa?
a. Hipertensi kronik
b. HT gestational
minggu
c. Preeklamsi
23.

>20

d. Eklampsi
e. HELLP syndrome

Kontrasepsi untuk mencegah PMS? Apa hayo?

a. Kontrasepsi hormonal
24.

b. Kondom

Tinggi fundus 1 minggu pasca ngelahirin?

a. 2
jari
di
bawah
umbilicus24 mnggu
b. 2 jari di atas simfisis

c. Setinggi umbilikus
d. 1/3 umbilikus-simfisis
e. Tidak teraba

25.
Ibu 45 tahun mengeluhkan ga mens 7 bulan, dan
mengkhawatirkan dirinya hamil. Ternlah di teyata setelah di tes,
haslnya ga hamil , iwayat amenorea sejak 13 atau 16 tahn yang lalu.
Pemeriksaan selanjutnya yang dilakukan?
a. LH n FSH
b. Biopsi endometrium

c. Histerosalfingografi
d. Hormon tiroid

e.
f.
3

g. SOAL UTAMA BLOK REPRODUKSI TAHUN 2012


1. Hormon untukhipertrofi alveolar payudara
a. Progesteron
2. Selain berat janin, kontribusi peningkatan BB terbesar
a. Uterus (4)
b. Amnion (5)
c. Plasenta (6)

d. Volume darah (3)


e. Komposisi lemak (2)

3. Perubahan fisiologis ibu hamil


a. EFC meningkat
b. TLC meningkat

c. Residu meningkat
d. Tidal meningkat

4. Pernyataan yg benar mengenai CO pada kehamilan adalah:


a. CO trimester 1 meningkat
karena HR meningkat
b. Peningkatan SV maksimal
pada trimester 3

c. HR
mningkat
seiring
bertambahnya kehamilan
d. SV meningkat 100%

5. Pernyataan apayang benar pada ibu hamil?


a. GFR meningkat
b. Ekskresi glukosa menurun

c. Kreatinin serum meningkat

6. Fisiologi kardiovaskuler pada bumil : penurunan SVR 5%


7. Apakah syaraf yang meneruskan nyeri ke cerebelum?
a. Thoraks 11
8. Nervus ereksi
a. N. kavernosus
9. Kerusakan pada pertemuan kedua labia minor pars medial apa??
a. Frenulum
b. Preputium
10.
11.

c. Clitoris

Wawan Eko Wahyudi


Hormonyang berperan dalam produksi ASI adalah

a. Oksitosin
b. Progestron

c. Estrogen
d. Prolaktin

12. Apakah pengaruh perbedaan perbandingan komposisi protein pada ASI


dan susu sapi terhadap kualitas dan pengaruh pada anak?
a.
b.
c.
d.

Protein yang lebih banyak lebih baik


Protein susu sapi sulit diserap sehingga terbuang
Anak dengan susu sapi lebih cerdas dan daya tahan tinggi
Protein lebih banyak membuat susu sapi lebih tahan lama
4

e. Sering menyebabkan diare


13.
14.

Muksan Abdul Somad


Uterotonika untuk terminasi kehamilan preeklampsia berat

a. Oksitosin drip 10 unit dalam


500
15.

b. Misoprostol
pembukaan

belum

ada

Terapi lain yang digunakan pada skenario di atas adalah :

a. Injeksi MGSO4 4mg IM


b. Dopamin

c. Dexamethasone harus 1
seri untuk pematangan
paru dulu

16. Gangguan hasrat seksual (Ibunya tidak bergairah, tapi bisa


melayani suamix) PP BLUES
17. Pengantin baru konsultasi karena merasa kurang puas terhadap
hubungan seksual mereka. Sudah melakukan foreplay, hub seksual cepat
berakhir, sehingga istri kurang puas.. Gangguan pada suami:
a. Disfungsi ereksi
b. Gangguan hasrat seksual

c. Ejakulasi primer
d. Nyeri saat senggama

18. Wanita dengan PMS, merasa dirinya memiliki emosi yang stabil, dapat
melakukan pekerjaan dengan baik, dan jika punya anak akan menjadi
ortu yang baik. Penyebab PMS adalah :
a. Perubahan hormon estrogen
dan progesteron
b. Perubahan
steroid
ovarium
19.

c. Karena ingin memilki anak


d. Penurunan
prolaktin,
peningkatan progesteron
e. Peningkatan progesteron

Hormon yang nyebabin PMS

a. Estrogen
b. Testosteron
20. Menurut kriteria DSM
Dismorphic Disorders?

c. Androgen
IV,yang

termasuk

kriteria

Premenstrual

21.
Insomsia, nafsu makan byk, labil, putus asa, depresi,
mudah tersinggung
22.

PMD muncul gejala kapan?

a. Fase folikuler
b. Luteal phase 1 mggu
sebelum mens

c. Post mens
d. Kapan aja boleh

23. Salah satu pilar safemotherhood ialah:KB, ANC berkualitas, APN,


perawatan pasca nifas
a. Pelayanan kebidanan
24.

b. Pelayanan
esensial

Penyebab tidak langsung kematian ibu adalah

a. Perdarahan post partum


b. Anemia
c. Infeksi
25.

obstetri

d. Eklampsia
e. Kekurangan
kronik

energi

Penyebab langsung kematian ibu hamil?

a. Perdrahan post partum


(45% terjadi pada 24 jam
pertama
pp,
68-73%
sminggu pp, 88% dalam 2
mggu pp)

b.
c.
d.
e.

Infeksi
Eklamsi
Anemis
Kekurangan energi kronis

26. Definisi obstetric social menurut WHO :ilmu yang mempelajari


hubungan
timbal
balik
reproduksi
manusia
dengan
lingkungannya
27. Ruang
lingkup
obsetri
socialsemnjak
ibu
merencanakn
kehamilan sampai ibu melahirkan
28. Tujuan khusus obsos : ndak jauh beda dari definisi
29. Seorang wanita 25 tahun dating ke dokter memeriksakan diri ke
praktek dokter dengan keluhan ingin merencanakan kehamilan. Di
anamnesis didapatkan wanita tersebut menikah sebulan yang lalu dan
ingin langsung hamil. Saat ini ia sedang haid hari ke 3 dari 7 hari lama
haid. Dari PF didapatkan pasien normal. Pada keadaan tersebut,
suplemen apa yang diberikan?
a. As. Folat
b. Ca
c. Fe

d. Vitamin A
e. Zink

30. Wanita, 37 tahun, G1P0, KU pusing, PF: TD 130/90 mmHg, N 80x/menit,


UK 24-26 minggu, Hb 10,5 gr/dl, protein +2.
31.

Pada kondisi tersebut bagaimana tatalaksana nutrisi yang tepat?

a. Asam folat
b. Ca
c. Fe

d. Vitamin A
e. Zink

32. Wanita hamil 10 minggu, BB 40, TB 150, harapan pertambahan berat


sampe selesai kehamilan (BMI = 17)
6

a. 12-18 kgjika BMI <19


b. 11-16 kgjika BMI 19-26
c. 7-11 kgjika BMI 26-29
33.

d. Kurang dari 7 kgjika BMI


>29

Mekanisme pertukaran O2 ibu-fetus

a. PO2 sinus maternal > PO2


fetus
b. PO2 perifer maternal < PO2
fetus
34.

c.
d. ..
e. Hb fetus merupakan HbF

Efek bohr ganda :

35.
Karena konsentrasi CO2 yang tinggi akan memiliki afinitas tinggi
terhadap Hb pula.
36.

Efek timbal balik saturasi oksigen fetus-maternal

37. Pada masa kehamilan, fetus laki-laki dapat mensekresi testosterone.


Sekresinya diakibatkan efek perangsangan sel-sel interstisial testis.
Hormone yang berperan
a. LH
b. Androgen
c. Progesterone
38.

d. Lutein
e. hCG buku merah
163 sama 166

hal

Tanda-tanda yang mengalami pre-eklamsi berat

a. TD 150/100 mmhg
b. Protein kuantitatif lebih dr
0,3g/L

c. Protein kualitatif 2+
d. Trombositosis

39. Kerusakan endotel, terjadi penurunan : prostasiklin


40. Antidotum MgSO4
a. Kalsium glukonas 10%
41. G3P2A0, 46 tahun, nyeri ulu hati, penglihatan kabur, TD 180/110
mmHg, protein +3. Diagnosis
a. Eklampsia
b. Preeclampsia berat
c. Preeclampsia ringan

d. Mola
e. Hipertensi kronik

42. Ibu perdarahan ballotement (-),DJJ(-), px HCG (+)diagnose


a. Mola hidatidosa
43. KPD penatalaksanaannya oleh dokter puskesmas : ampicilin 1gr/6
jam IV rujuk
44. Dateng ngecek kehamilan TD 150/100 mmHg, protein positif 2.
Diagnosis Preeklamsi Ringan
45. Penatalaksanaan apa yang dilakukan untuk preeklampsia ringan?
7

a. Konservatif
b. MgSO4 eklampsia
c. Nifedipinhipertensi
kronik/gestasional
46.

d. Kalsium
glukonasantidotum mgso4
e. Labetololkontraindikasi
pada bumil

Kapan ibu kontrol tekanan darahnya?

a. 1 minggu kemudian
b. 2 minggu kemudian

c. 3 minggu kemudian
d. 4 minggu kemudian

47. G3P2A0, mual muntah, nyeri ulu hati, nyeri kepala, UK 15 minggu, TD
180/120 mmHg, protein +3, obat anti-HT (PEB) :
a. Hidralazin
b. Labetalol
c. Metildopa

d. Diazepam
e. Nifedipin 2x250 mg

48. Ndak tau sapa


49. Apriana Aidiatul Fitri
50. Haid tak tratur, darah, benjolan kenyal, sebesar hamil 8 minggu,
diagnosis
a. Mioma

b. Miosarkoma

51. Perempuan 27 tahun nyeri hebat pada perut kanan bawah disertai
darah coklat tua, telat haid 2 bulan. Lemah, pucat, 90/70 mmHg, N 120,
RR 24x. Cavum Douglas menonjol, massa pada adnexa kanan, uterus
sebesar telur bebek, nyeri goyang servix. Diagnosis
a. Abortus insipiens
b. Abortus Iminens
c. KET buku merah
479

d. Torsi kista ovary


e. Peny. Inflamasi pelvic ada
demam

hal

52. Ibu mengalami keguguran terus menerus. Anaknya jg mati di dalam


rahim, pemeriksaan laboratorium apa yang dilakukan?
53.
Antara C. TSH, Progesteron sama estrogen atau D. Px autoimun
antisperma
54.

Diagnosis :

55.
Pokoknya bingung antara kala I MACET, kala I MEMANJANG, ato
kala I KASEP
56.

Tindakan dokter puskesmas pada no 49 :

a. Langsung rujuk karena KPD


> 12 jam

b. Resusitasi
IU,antibiotik,rujuk
c. Antibiotik,rujuk
8

d. Antibiotik,observasi
jam,rujuk
57.
58.

e. Antibiotik,akslerasi
oksitosin,rujuk

Penanganan :
Metotreksat, mekanisme kerja

a. Memblok
konversi
as.folat
menjadi
tetrahidrofolat
b. Mengganggu sintesis purin
59.

c. Sebagai antagonis reseptor


androgen
d. Antagonis estrogen
e. Inhibisi oksidasi xantin

ANC: fundus kepala, VT bokong. Diagnosis

a. Inersia uteri primer


b. Inersia uteri sekunder
c. Hipertonik

d. Sungsang
e. Lintang

60. Maless
61. Pasien mengeluh perdarahan, nyeri perut, ukuran uterus sesuai
kehamilan, uterus tidak tegang (karena masih sering kontraksi).
Diagnosis
a. KET
b. Solusio plasenta

c. Abortus insipiens

62. Wanita datang dgn kehamilan 37 mggu, HIS tdk adekuat.


Penatalaksanaan ?
63. Maless
64. Maless
65. Post SC,payudara keras dan membesar : bendungan payudara
66. Perut tegang, keluar lokhia metritis
67. Ibu,post partum 4 minggu,mengeluh payudara menegangdan bengkak.
Pada payudara kiri tampak kemerahan dengan batas tegas. Diagnosis
Anda
a. Abses pelvis
b. Deep vein trombosis
c. Mastitis

d. Metritis
e. Bendungan Payudara

68. Wanita 24 tahun post partum hati ke-7, badan demam terus-menerus,
nyeri perut bagian bawah, riwayat lahir di dukun, px perut bagian bawah
besar dan px dalam didapatkan pembengkakan adneksa dan kavum
Douglas. 2 hari yang lalu konsumsi antibiotic yang dianjurkan dokter.
Diagnosis
a. Abses pelvis
b. Deep vein thrombosis
c. Bendungan payudara

d. Mastitis
e. Metritis
9

69.
70.

Deep vein trombosis


Ruptur perineum grade 2 :

a. Selaput lendir vagina


b. Kulit perineum
c. Otot perineum
71.

d. Sphingter ani eksterna


e. Mukosa rectum

Akibat robekan perineum derajat IV :

a. Inkontinensia urin
b. Inkontinensia alvi kencing

c. Fistula
d. Inkontinensia ani

72. Skenario panjang, ibu hamil..Indikasi persalinan yangpaling tepat


adalah
a. Ergometrin
b. Prostaglandin
73.

c. Oksitosin
d. Kina

Kerja ergometrin

a. Dipengaruhi
hormon
estrogen/progesterone
b. Merelaksasi cerviks(tidak),
hentikan perdarahan,
74.
75.

c. Diberikan untuk provokatus


medisinialis
d. Kontraksi tanpa relaksasi
uterus

Efek ergometrin : bikin tetani


Endometrium ore, anemia def. besi. Diagnosis

a. Ca endometrium

b. ..

76. Wanita 38 tahun, 2 bulan kemarin menjalani kuretase karena abortus


inkomplit. Datang ke puskesmas karena ada bercak darah keluar dari
vagina. Pada pemeriksaan, vital sign dbn, mual-mual,serviks membesar
segenggaman orang dewasa dengan konsistensi lunak, fluxus (+),tes
kehamilan (+). Diagnosis yang mungkin : PTG (penyakit tropoblas
ganas) atau koriokarsinoma
77. Tindakan yang dilakukan untuk nomer 70, ket: ibu pernah abortus
incomplete, sekarang spotting keluhannya, fluxus (+)
a. Dilatasi + kuret
b. Histerektomi
kemoterapi

c. Histerektomi +tubektomi
d. Progesterone oral

78. What's the most diagnose on patient with hirsutism, acne


a. polycistic ovarian disease
79. Yang paling memungkinkan penyebab adnexal massa adalah
a. Fokus nekrosis
b. Maturasi folikel

c. Kista ovarium benigna


d. Kista ovarium maligna
10

80. Cewek 15 tahun, pernah mens, terus mens nyeri, sampai tidak masuk
sekolah 1-2 hari. Diagnosis
a. Endometriosis
81. Ball massa itu apa pokoknya gitu dah Leimioma
82. Tempat tumor sering terjadi?
a. Intramural uterus
b. Submucosa serviks

c. Intramural serviks

83. Multiple Chocolate brown colored 2 mm


a. Endometriosis
84. Ibu hmil + mioma, kontraspsi
a. Mini pil
b. IUD
c. Norplant

d. Pil kombinasi
e. MOB metode
billing

ovulasi

85. Administration of which the following hormones might have prevented


this patient condition? (scenario 78)
a. FSH
b. hCG
c. Insulin

d. Progesterone
e. Testosterone

86. Tumor marker untuk screening karsinoma ovarium?? CA 125


87. Wanita punya 2 pasangan seksual aktif, nyeri pelvis. PF serviks motion
tenderness, adneksal tenderness. Penunjang leukositosis, Hct meningkat
(?), hCG (-), urinalisis (-). Diagnosis
a. Ectopic pregnancy
b. PID

c. UTI

88. Mekanisme kerja doksisiklin : menghambat sintesis protein


89. Efek doksisiklin (turunan tetrasiklin) pada struktur fetus
a. Jantung
b. Ginjal
c. Limbs

d. Gigi
e. Tendon

90. Wanita P3A1 pernah KET 1 bulan yang lalu, ingin menggunakan KB 3th,
suami tidak ingin kondom atau pantang berlaka, KB yang efektif adalah :
a. IUD
b. KB depoponover injeksi

c. Implanharus
6
pasca persalinan
d. Pil kombinasi

mggu

91. Suami tidak mau coitus interuptus, ibu tidak mau pake kalender?
kondom
11

92. Hari 11 koitus tanpa kondom,mekanisme kerja KB(kondar estrogen


dosis tinggi)?
a. Hambat FSH
b. Peradangan endometrium

c. Hambat LH
d. Tambah kekentalan serviks

93. Kontrasepsi mantap, 100% tidak ingin hamil, usia cukup, anak cukup,
tidak mengganggu libido
a. Bedah (vasektomi, tubektomi)
94. Maless
95. Pasien pake AKDR 1 bulan yang lalu, apa penatalaksanaan? Ibuprofen
dulu baru cabut
96. Pasangan datang konsultasi belum punya anak, istri sempet make KB
suntik sebulan, haid gak teratur, obat yang dikasi untuk induksi ovulasi
dan meperbaiki siklus
a. Klomifen
97. Perdarahan pervaginam, hamil usia 15 mggu, tanpa rasa nyeri, porsio
tertutup,TFU 19, DJJ (-), ballotement (-), ada fluxus. Diagnosis
a. Mola hidatidosa
b. Abortus iminens

c. Abortus iminens
kista ovarii

suspect

98. Penatalaksanaan nomer 91 adalah? Kuretase


99. Manajemen abortus provokatus
a. Prostaglandin oral
100. Ibu hamil, asma, tatalaksana ? betas 2 agonis kerja cepat +
kortikosteroid
101. Seorang wanita 25 tahun,belum memlki anak, haid tidak teratur, nyeri
perut saat haid, haid datang 4 bulan kemudian, muncul jerawat, IMT 28
kg/m2, diagnosisnya apa?
a. PCOS
b. Endometriosis
c. Endometritis

d. Gangguan haid
e. Disminore

102. Kriteria minor dari penyakit diatas(diagnosis 1 mayor, 2 minor)


a. Akne
b. BMI, resistensi insulin,
LH/FSH dan lh lebih dari
2,5, hirsutisme

c. Haidtakteratur
d.
Mayor : anopulasi dan
hiperandrogenisme
(hirsutisem, akne)

103. Wanita, KU nyeri hebat menjelang haid. Sudah 2 tahun menikah belum
punya anak. Diagnosis infertilitas disetai dugaan
a. Endometriosis
b. Endometritis
c. PCOS

d. Agenesis uteri
e. Disminore
12

104. Pada kasus nyeri haid + infertil 2 tahun, teori apa yang mungkin
terjadi?
a. Retrograde
105. Px penunjang endometriosis : Laparoskopy
106. Gangguan haid, kelaianan payudara, kelainan dimana ? Gangguan
hipotalamus hipofisis
107.
108.

109.

SOAL UJIAN BLOK XVI ANGKATAN 2008


110.

1. Saraf apa yang menghantarkanrasa sakit dari uterus ke serebrum ?


a) pleksus frankhouse
c) saraf sakral 1
b) saraf torakalis 11
d) saraf pudendus
2. Ligamentum yang ikut terangkat saat operasi kista ovarium, yang terletak
di atas uterus dan di antara tuba dan ovarium adalah
a) ligamentum cardinal disebut jg lateral servikal (mnuju lateral untuk
brsatu dengan fascia atas diafragma pelvis)
b) ligamentum sakrum ndk da, biasanya sakroiliac, sakrospinale,
sakrotberale
c) ligamentum sakrovaginam di dorland ndk da
d) ligamentum latum
e) ligamentum pubovesicalis
3. jawaban: ndak jelas, kemungkinan ligamentum cardinal.
ligamentum cardinal merupakan ligamentum yang terpenting untuk
mencegah uterus turun. Ligamentum ini berjalan dari arah srviks dan
puncak vagina ke arah lateral pelvis.
Ligamentum sakrum merupakan ligamentum yang berjalan dari serviks
bagian belakang ke arah os sakrum.
Ligamentum sakrovaginam merupakan ligamentum yang berjalan dari
os sakrum ke arah vagina.
Ligamentum servikalis merupakan ligamentum yang berjalan di arah
serviks ke uterus.
Ligamentum puboservikalis yang berjalan dari pubik atau fundus
uterus ke arah serviks.
Ligamentum latum yang menyelimuti tuba ke arah uterus
Ligamentum rotundum yang menahan uterus ke arah inguinal.
4. Sumber: buku ilmu kebidanan sarwono halaman 122
5.
6. Nervus yang memicu da menyebabkan ereksi pada lak-laki adalah
a) N. cavernosus
b) N. T3-T5
c) N. pudendus
d) N. dorsalis penis
13

7. innervasi :
8. - n.dorsalis penis (dorsum & glans penis)
9. - n.ilioinguinalis (kulit radix penis)
10.

- r.profundus n.perinealis (urethra)

11.

- n.cavernosus (ereksi)

12. Montgomery tubercles adalah buku tramed anc, papil-papil kecil


pada areola payudara (glandula areola) yg pada khamilan trlihat
mmbesar (tubercle montgomery) (sbg tanda mungkin khamilan)
13. Perubahan kardiovaskuler pada waita hamil adalah ...
a) penurunan cardiac output
d) penurunan
resistensi
b) peningkatan diastolik
perifer
c) peningkatan sistolik
e) peningkatan
resistensi
pulmonal
f)
g) perubahan Cardiovascular system (william obstetri halaman 196):
peningkatan denyut nadikarena difragma semakin terangkat
sehingga jantung bergeser ke kiri dan atas
cardiac ouput meningkat karena terjadi penurunan resistensi
vaskuler.
Penurunan resistensi pulmonal dan vaskuler juga terjadi karena
dicurigai hormon estrogen mempengaruhi reseptor angiotensin.
diastolik dan sistolik normal karena yang mengkompensasi adalah
pembesaran dari otot jantung.
Penurunan tekanan osmotik koloid.
14. Yang berperan dalam pemberian nutrisi embrio selamamasa kehamilan
awal ...
a) Proliferasi sel endometrium oleh estrogen
b) Pembesaran sel stroma oleh progesteron
c) Peran hCG dalam perubahan endometrium menjadi sel desidua
d) Kerja estrogn dan progesteron yang sinergis
e)
f) Yang berperan dalam peningktan sel stroma adalah progesteron.
Namun perlu diketahui progesteron dihasilkkan oleh korpus
luteum. Selama awal kehamilan HcG berperan untuk mencegah
proses degenerasi dari korpus luteum sehingga memungkinkan
untk tetap mensekresikan progesteron yang berfungsi dalam
pemberian nutrisi embrio. (william obstetri)
7. Keadaan yang memungkinkan transfer O2 adalah ...
a) pO2 sinus maternal >
c) Difusi sederhana karena
fetus
gradien tekanan rata-rata
b) Difusi sederhana karena
50 mmHg
pO2 fetus > ibu
14

d) Konsentrasi Hb fetus >


e) Hb fetus adalah HbF
ibu
f)
g)
Penyaluran oksigen dibatasi oleh aliran darah. Oksigen dari
darah ibu terus menerus mengalir diruang antar-vilus ke janin, saturasi
darah di janin hampir sama dengan saturasi darah di kapiler ibu.
Saturasi oksigen darah vena umbilikalis juga setara , tetapi dengan
tekanan parsial oksigen sedikit lebih rendah. (william halaman
148)
8. Keadaan yang terkait dengan efek Bohr (konsentrasi co2 dan ion hidrogen
yg tinggi, seperti yg ditemukan pada kapiler dalam jaringan yg aktif
secara metabolik, menurunkan afinitas hb trhadap o2 sehingga kurva
disosiasi o2 bergeser ke kanan : dorland) ganda?
h) Lebih banyak oksigen mengalir dari darah ibu ke fetus
i) Jawaban: efek bohr merupakan efek penurunan afinitas oksigen
terhadap hemoglobin akibat dari peningkatan CO2 di dalam darah
sehingga co2 akan diangkut kemudian dibawa ke paru. Pada janin,
tekanan parsial co2 sedikit lebih tinggi daripada yang terdapat di plasenta
sehingga ini membuat co2 berikatan dengan hemoglobin yang kemudian
dibawa keluar ke plasenta dan sirkulasi ibu. Sedangkan tekanan parsial
oksigen di sirkulasi ibu yang menuju plasenta
tinggi dibandingkan
dengan janin dan plasenta, sehingga oksigen mengalir dari darah ibu ke
janin. Hal yang mempengaruhi afinitas oksigen selain co2 adalah suhu
dan pH darah.
9. Kosong
10. Kosong
11. Seorang wanita dengan usia kehamilan 8 minggu menunjukkan hasil
USG blinded ovum, dokter memutuskan melakukan abortus provokatus
dengan ...
a) businasi
d) ergometrin oral
b) kina IV
e) prostaglandin oral
c) oksitosin
f)
g) blighted ovum atau dikenal sebagai kehamilan anembrionik
merupakan suatu kehamilan patologik dimana mudigah tidak terbentuk
sejak awal walaupun kantong gestasi tetap terbentuk. Di samping
mudigah kantomg kuning telur juga tidak ikut terbentuk. Pada
pemeriksaan USG gambaran ini diperlihatkan pada usia kehamilan 7-8
minggu. Pengelolaan kehamilan anembrionik dilakukan terminasi
kehamilan dengan dilatasi dan kuretasi secara elektif. Untuk membuat
dilatasi serviks diberikan prostaglandin yang dimasukkan ke vagina dalam
bentuk supositoria. Namun beberapa penelitian juga membuktikan
prostaglandin oral juga efektif dalam abortus pada gestasi dini. Padaa
kehamilan trimester kedua, obat yang banyak digunakan untuk abortus
adalah
oksitosin,
Sedangkan
ergometrin
digunakan
dalam
15

penatalaksanaan perdarahan post partum atau retensio plasenta. (obgin


william halaman 973)
12. Seorang wanita, usia 20 tahun, merasa nyeri perut, tapi tidak ada
perdarahan. Pasien sering mengeluh pusing, vital sign dalam batas
normal. Pemeriksaan abdomen kotraksi positif. Penatalaksanaan yang
diberikan ?
a) Asam mefenamat
c) MgSO4
b) Nifedipin
d) Terbutalin
13. Wanita, 25 th, G1P0A0, his 1x/10 menit selama 15 detik, posisi kepala
4/5, belum ada pembukaan. Rencana induksi persalinan yang diberikan
a) ergometrin
d) kina
b) prostaglandin
e) kalsium glukonas
c) oksitosin
f)
g)
pada skenario diatas pematangan serviks belum matang. Untuk
memilih proses induksi persalinan dengan bedah atau farmako harus
dihitung dulu bishop skornya. Namun jika yang dipilih adalah tindakan
farmako seperti pada skenario diatas, serviks harus dimatangkan
terlebih dahulu. Pada kasus diatas his juga belum adekuat dan belum
ada pembukaan. Upaya Farmakologis awal untuk mematangkan serviks
berpusat pada pemakaian preparat prostaglandin (dinoproston, E2)
intraservikal atau intravaginal. Namun skrg metode telah dikembangkan
secara oral. (obgin william halaman 521)
14. Kosong
15. G2P1A0, keputihan sejak 2 minggu sebelum hamil, tampak bintil-bintil
merah di vagina. Tatalaksananya yang diberikan
a) ketokonazol oral
d) metronidazole
per
b) nistatin per vaginam
vaginam
c) ciprofloksasin oral
e)
f)
kemungkinan dikarenakan infeksi dari trichomonas, jadi
pengobatannya dapat dengan metronidazol. Gambaran klinik dari
trichomonas adalah strawberry serviks yang berupa makularis di daerah
serviks. Vaginitis ditandai cairan kuning, berbau, dan pruritis vulva.
16. Obat anti hipertensi yang cara kerjanya meningkatkan curah jantung
dan menurunkan resistensi perifer adalah
a) labetolol
c) hidralazin
b) nifedipin
d) Mg2SO4
17. Kosong
18. Wanita 35 tahun, G2P1A0, hamil 15 minggu, dengan perdarahan, tidak
ada nyeri, tanda vital normal, TFU 19 cm, DJJ tidak ada, ballotement tidak
ada, OUE tertutup, diagnosisnya ...
a) abortus imminens
b) KJDR
c) mola hidatidosa (masa besar vilus korion yg mmbngkak, dan scara
makros tampak seperti anggur. Ada yg komplit dan parsial, komplit
16

tdk mmngkinkan trjadinya embriogenesis shingga tdk prnah


mngandung bagian janin, semua vilus korion abnormal, sel epitel
korion diploid. Parsial masih mmungkinkan pmbntukan mudigah awal
shingga mngandung bagian-bagian janin, memiliki bbrapa vilus
korion normal, hampir slalu triploid. Trjadi pada umur <20 dan >40
thn. Dtmukan pada gestasi mgg 12 -14 dimana gestasi trlalu besar
untuk usianya. Dx usg, pnigkatan hcg, tdk adanya bag janin dan djj)
robin kumar hal 785.
d) abortus imminens dengan kista ovarium
e) antepartum bleeding
19.
mola hidatosa adalah suatu kehamilan yang berkembang tidak
wajar dimana tidak ditemukan janin dan hampir seluruh vili korionik
mengalami perubahan berupa degenerasi hidropik. Gejalanya tidak
berbeda dengan kehamilan awal berupa mual muntah dan pusing.
Uterus juga lebih besar dibandingkan dengan umur kehamilan. Pasien
juga mengalami oerdarahan. Pada USG gambaran khas mola adalah
seperti badai salju atau sarang lebah. Lalu tanda kehamilan lain tidak
ditemukan seperti ballotement dan detak jantung janin. Terjadi
peninggian HcG pada hari ke 100 yang sangat sugestif. (buku kebidanan
sarwono halaman 488-489)
20. Wanita usia kehamilan 15 minggu, perdarahan, tidak nyeri,OUE masih
tertutup, pemeriksaan yang sebaiknya dilakukan adalah
a) USG
c) Pemeriksaan lainnya
b) HCG
21. Kosong
22. Terapi untuk pengobatan kasus diatas adalah
a) Kemoterapi
b) Histerektomi dilanjutkan dengan kemoterapi
c) Kemoterapi dilanjutkan dengan tubektomi
d) Kemoterapi dan radiasi
e) Pemberian progesteron untuk mempertahankan kehamilan
23. Kosong
24. Kosong
25. Perempuan 25 tahun, datang dengan perdarahan pervaginam dan
merasakan mules.Pada pemeriksaan dalam didapatkan pembukaan porsio
(+)(inkomplit atau insipiens), teraba jaringan, vital sign dan fundus
uteri normal. Diagnosisnya
a) abortus iminens : merupakan perdarahan pervaginam pada usia
kehmilan kurang dari 20 minggu , ostium uteri masih tertutuo dan
hasil konsepsi masih baik di dalam kandungan. Umur kehamilan dan
tes kehamilan masih positif.
b) abortus insipiens: merupakan abortus yang sedang mengancam yang
ditandai dengan serviks yang sudah mendatar dan ostium urteri yang
sudah membuka, akan tetapi hasil konsepsi masih di dalam kavum
uteri dan dalam proses pengeluaran.
17

c) abortus incomplet: sebagian hasil konsepsi telah keluar, dan


kanalis serviks masih membuka, namun masih ada sebagian
hasil konsepsi yang tersisa di dalam. Terjadi pada umur
kehamilan kurang dari 20 minggu atau berat janin kurang
dari 500 gram
d) abortus complet: hasil konsepsi lengkap keluar. Ostium telah
menutup, besar uterus tidak sesuai dengan umur kehamilan,
biasanya hanay diberikan roborantia, tidak uterotonika.
26. Komplikasi yang sering terjadi pada 6 hari setelah masa prepurium
adalah
a) sistitis
nonkomplikata,
tetapi
b) pielonefritis
tetap
merupakan
c) infeksi luka jalan lahir
masalah
besar
pada
d) endometritis
(infeksi
cesar) obs william hal
uterus
jarang
terjadi
745.
setelah
persalinan
e) Mastitis
pervaginam
27. Hamil 12 minggu, nasehat yang diberikan pada trimester 2 adalah
a) tanda-tanda persalinan
b) tanda-tanda hipertensi pada kehamilan
c) perencanaan persalinan
28. Ibu datang antepartum usia kehamlan 36 minggu, perdarahan,
didapatkan tanda-tanda fetal distres, hipertensi, dan perut
menegang. Diagnosisnnya
a) plasenta previa
c) vasa previa
b) solusio plasenta
d) robekan jalan lahir
e)
f) plasenta previa adalah kelainan letak plasenta yang menutupi jalan
lahir sehingga menimbulkan perdarahan. Terjadi pada wanita dengan
paritas tinggi. Ciri yang menonjol dari plasenta previa yaitu perdarahan
uterus yang keluar melalui vagina tanpa rasa nyeri. Perdarahan biasanya
terjadi pada akhir trimester kedua keatas. Perdarahan tidak banyak dan
berhenti sendiri naamun berulang tanpa sebab yang jelas (buku sarwono
kebidananan halaman 497)
g)
h) solusio plasenta memilki gejala dan tanda klinik yang klasik yaitu
perdarahan yang berwarna tua keluar melalui vaginna, rasa nyeri perut
dan uterus tegang terus menerus mirrip dengan his partus prematur.
(buku kebidanan halaman 507).
29. Kosong
30. Wanita usia 18 tahun, menarche 15 tahun, tiap haid terasa nyeri, tidak
ditemukan adanya massa. Apa yang dilakukan?
a) observasi tiap bulan
d) laparoskopi diagnostik
b) observasi 3 bulan
e) Mioma uteri
c) terapi hormonal
f) Bisa juga curiga ca ovarium dx umur <20 dan >60
18

31. HPHT 04-08-2010. Taksiran persalinannya 11 Mei 2011


g) H+7 b-3 tahunnya trgantung bulan hpht
32. Pemeriksaan penunjang apa yang dilakukan perihal persiapan
persalinan ?
a) lab rutin, Hb, urinalisis, GD
c) CT/BT
2 jam/PP
d) HbsAg hepatitis b
b) USG
33. Wanita, usia kehamilan 32-34 minggu, datang dengan keluhan
perdarahan setelah koitus, TD 180/110, darah yang keluar berwarna
hitam lebih dari 50 cc, proteinuria positif 2, terjadi karena apa?
a) kelainan
perlekatan
b) preeklampsia berat
plasenta : kemungkinan
c) trauma
solusio plasenta dengan
d) koitus
faktor
predisposisi
preeklmapsia
e)
f) Klasifikasi solusio :
g) Ringan lepas 25%, sedang 25-33%, berat >33%
h) Gejala : prdarahan prvaginam warna khitaman, nyeri perut kontinu
(prlahan to mndadak), defans muskular, palpasi bag janin sulit teraba,
anemia to syok tdk sesuai dngan prdrahan yg kluar prvaginam, gangguan
hemostatis,
34. Pengelolaan pada pasien di atas adalah ...
a) Pemberian MgSO4, nifedipin, amniotomi
b) Pemberian MgSO4, nifedipin, drip oksitosin
c) Pemberian MgSO4, nifedipin, baring miring, O2
d) Pemberian MgSO4, nifedipin, grojol RL, baring miring, O2
e) Pemberian MgSO4, nifedipin, rujukan
35. Tumor ovarium yang sudah mengalami keganasan sehingga
mengalami perlukaan di sekitarnya,tindakan apa yang harus dilakukan?
a) histerektomi radikal
c) debulking
b) pan histerektomi???????
d)
e) intervensi bedah untuk kanker ovarium adalah histerektomi abdominal
total, salpingo-ooforektomi bilateral dngan omentektomi, eksplorasi
abdominal lngkap dan biopsi multipel peritoneum, kelenjar aortik dan
kelenjar pelvis. Pngobatan lain kemo, radiasi atau kombinasi. Price wilson
hal 1298
f) pd prinsipnya tumor ovarium mmbutuhkan pmbedahan, tetapi ada
beberapa kista bnigna yg umumnya tdk mmbtuhkan pmbdahan sprti
kista folikel de graaf, kista korpus luteum, dan kista endometrium.
g)Tumor ganasovarium pd tingkat klinis 1 dan 2 dilakukan pmbedahan
dasar dngan pngangkatan uterus, adneksa omentum dan apendiks. Pd
tingkat 3 dan 4 dilakukan pmbedahan dasar ditambah dngan
pngangkatan tumor sbanyak mungkin yg disebut bulkektomie.
19

h) 1 terbatas pd ovarium, 2 trbatas pd pelvis, 3 mnyebar ke intra


abdominal, 4 kluar abdomen. De jong hal 732
36. Wanita umu 18 tahun, keluhan tidak pernah mens, tiap bulan perasaan
sakit pert, pada pemeriksaan fisik ada tonjolan pada selaput vagina
a) himen imperforata
b) kista ovarium
c)
d)
PCOS : amenore, masa repro, jerawat, akantosis, obese
e)
Hymen imperforata : amenore, edema pada vagina eksterna
f)
Agenesis (sindrom rokitansky hauser) vagina : mnyempitnya
vagina tanpa uterus namun rambut pubis normal
37. Kosong
g)
38. Ibu 25 tahun, bayi 2 bulan, ingin menggunakan kontrasepsi, dia sudah
menggunakan ASI ekslusif. Apa kontrasepsi yang dianjurkan untuk ibu ini

a) injeksi DMP
c) MAL
b) implan
39. Wanita 23 tahun, memiliki anak berusia 4 bulan, sekarang anak
menderita DHF selama perawatan tidak menyusui. Ibu selama ini
memberikan ASI ekslusif belum haid dan belum menggunaan kontrasepsi
yang lain. Kapan ibu memulai kontrasepsi ?
a) Menggunakan MAL lanjutkan ASI ekslusif
b) Menggunakan KB setelah haid
c) Langsung menggunakan KB saat itu juga
d) Stop menyusui, langsung menggunakan KB
40. Wanita 20 tahun, G1P0A0 his 3 kali/10 menit, pembukaan 8 cm. Kapan
pemeriksaaan dalam ulang dilakukan ?
a) 30 menit
c) 4 jam
b) 2 jam
d) 6 jam
e)
f)
Priksa tiap 30 menit buat kontraksi, tiap 4 jam buat vt klo da
pnyulit lebih sering, semua fase pembukaan servik.
41. G1P0, kehamilan gemmeli, anak pertama presentasi kepala, anak
kedua presentasi sungsang. Rencana persalinan ?
a) pervaginam
c) induksi oksitosin
b) SC
d) observasi 2 jam
42. Wanita, 28 tahun datang untuk mengganti kontrasepsi. Sebelumnya
menggunakan suntikan 3 bulan tapi mengeluh BB meningkat. Kemudian
dia memilih menggunkan pil kombinasi. Jika lupa minum pil bagaimana?
a) minum pil saat ingat
d) minum pil aktif
b) minum 2 pil sekaligus
e) minum pil tidak aktif
c) minum pil dengan paket
baru
43. Wanita 20 tahun G1P0A0, tidak pernah ANC, tanda bersalin sejak
beberapa hari yang lalu, ditolong oleh dukun, tanda vital baik, TFU 32cm,
20

DJJ normal, his 1x/10 menit, masing-masing 30 detik, penurunan kepala


4/5, konjugata vera 6 cm. Apa yang dilakukan?SC karena CPD
a. Pelvic Inletatau Pintu Atas Panggul (PAP)
Merupakan bidang yang dibentuk oleh promontorium os sacral 1, linea
innominata (terminalis), dan pinggir atas simfisis.
Terdapat empat diameter pada PAP yaitu diameter anteroposterior,
diameter transversa, dan 2 diameter oblique.
- Konjugata vera (jarak antara tepi atas simfisis dengan
promontoriun) = 11 cm.
- Diameter transversa (jarak terjauh garis melintang PAP) = 12,5
-13 cm.
- Konjugata oblique (jarak dari tengah simfisis bagian dalam ke
promontorium) yang merupakan konjugata obstetrika yang
paling penting.
Normalnya 10 cm. Konjugata obstetrik
merupakan diameter anteroposterior yang terpendek dimana
kepala janin harus melaluinya.Diameter konjugata oblique tidak
dapat diukur secara langsung, tetapi dengan terlebih dahulu
mengukur diameter konjugata diagonalis. Diameter konjugata
diagonalis diukur dari jarak antara promontorium dengan batas
bawah simfisis.
- Konjugata obstetrik = konjugata diagonalis dikurangi 1,5/2 cm.
44. Wanita keputihan, cairan kekuningan, nyeri, serviks kemerahan,
diagnosisya ...
a) GO
c) Trichomonas
b) Clamidia
vaginalisbintik kemerahan
trachomatisamis
d) Bakterial vaginitis
45. Penyebab displasia sedang ?
a) HIV
c) GO
b) HPV
46. Kosong,,
47. Pembukaan awal 3 cm kemudian menjadi 5 cm. Tahap apa?
a) akselerasi fase laten kala 1
d) maksimum slope fase
b) deselerasi fase laten kala 1
aktif kala 1
c) akselerasi fase aktif kala 1
48. Apa yang dimaksud dengan engagement?Diameter Biparietal
49. Wanita nulipara, bersalin selama 3 jam, bayi 3600 gram, luka jalan
lahir, dilakukan anestasi kemudian dijahit. Sampai sekarang belum bisa
miksi,apa yang menyebabkan keadaan tersebut?
a) Anestesi lokal
b) persalinan lama : kala dua memanjang dibatasi 2 jam dan
diperpanjang smpe 3 jam apabila mnggunakan analgesik regional
pada nulipara, multi sejam dan 2 jam. Komplikasi kala 2 lama :
infeksi intrapartum, ruptur uteri, cincin retraksi patologis, pmbntukan
fistula (vesikovaginal, vesikoservikal, rektovagina), cedera otot dasar
panggul, pd janin kaput suksadeneum, molase kepala janin
21

c) BB bayi 3600 gram


d) nulipara
50. Kosong
51. Wanita 26 tahun, 7 minggu yang lalu melahirkan dan kini menglami
dispareuni. Penyebab dispareuni adalah ...
a) tingginya kadar oksitosin
c) hematoma vulva
b) rendahnya
kadar
d) laserasi jalan lahir belum
estrogen
sembuh sempurna
52. Perubahan respirasi apa yang terjadi pada saat masa nifas?
a) penurunan volume total
b) penurunan volume tidal
paru
paru
53. Kosong
54. Kosong
55. Sakit perut hilang timbul, hisnya 3x/10 menit durasi 45
detik,penurunan kepala 3/5, pembukaan 4 cm. Evaluasi 4 jam,
pembukaan masih tetap 4 cm, hisnya 2x/10 menit durasi 25 detik.
Tindakan yang dilakukan ?
a) SC
d) oksitosin drip
b) vacum
e) evaluasi 4 jam
c) forceps
56. Penanganan preeklampsia berat MGSo4 IV 4 gr 5 menit6gr RL
2gr iv 10 ml
57. Wanita hamil 37 minggu, datang dengan keluhan pusing 2 hari yang
lalu, dan nyeri ulu hati, mual muntah, pada pemeriksaan fisik ada edema
tungkai dan mata, heart rate 100x/menit, RR 30x/menit, TD 160/110
mmhg, janin letak sungsang, DJJ 130x/menit. Hasil pemeriksaan labyang
diharapkan agar dapat menegakkan diagnosis ...
a) protein < 2 gram kualitatif tidak terdeteksi, kuantitatif 10150 mg / 24 jam
b) bilirubin 2 mg total 0,2-1,2 mg/dl direk 0-0,4 mg/dl
c) SGOT 32 = ast 5-40 IU/L sgpt=alt 0-40 IU/L
d) trombosit > 100.000
e) asam urat 7 mg 10-59 thn p 2-8 mg/dl l 2,5-9 mg/dl
58. Wanita hamil 37 miggu, TD 160/100 mmHg, pusing, nyeri ulu hati,
janin letak sungsang, DJJ 130x/menit, tindakan awal ?
a) MgSO4 IM
d) tirah baring kiri
b) antasida
e) IV D5%
c) analgetik
59. Indikasi terminasi kehamilan pada kasus?
a) berat janin 3000 gram
c) DJJ 140x/menit
b) umur
kehamilan
37
minggu
60. Wanita, hamil 2 bulan, mengeluh perdarahan, awalnya tidak dirasakan
nyeri, kemudian istirahat. Beberapa jam kemudian pasien merasa
perutnya mulas dan ada perdarahan. Diagnosis yang tepat pada kasus di
atas adalah
22

a) abortus inspiens
d) mola hidatidosa
b) abortus iminens
e) KET
c) abortus inkomplet
61. Kosong
62. Keamanan USG adalah
a) Gelombang
d) Tidak
menyebabkan
elektromagnetik yang aman
kecacatan
b) Gelombang sifatnya pulsatif
e) Memakai gelombang suara
dengan dosis yang aman
frekuensi rendah
c) Tidak invasif
63. Wanita, 2 hari postpartum, tidak menyusui anaknya, datang dengan
keluhan payudara nyeri dan bengkak. Tatalaksana yang sesuai ?
a) dikompres
dan
dibebat
c) berikan
bromokriptin
(mngurangi hiperprolaktin)
b) ASI diperas
64. Kontrasepsi yang mengurangi produksi ASI
a) IUD
b) Progestin only pil
c) Depo Provera (hanya progesteron pd injeksi) klo kombinasi sama
estrogen disebut siklofom
d) Pil kombinasi
65. Kosong
66. Kanker yang berhubungan dengan kontrasepsi hormonal kombinasi
a) kanker payudara
c) kanker endometrium
b) kanker ovarium
d)
e)

PIL KOMBINASI

f)

Efek kerja

g)

Mencegah terjadinya ovulasi


Mencegah implantasi
Mengurangi dan mengentalkan lendir serviks sehingga sperma tidak
dapat masuk uterus
Perubahan pada motilitas tuba fallopii dan uterus sehingga
transportasi telur akan terganggu
Indikasi

h)
Pada prinsipnya hamper semua ibu boleh menggunakan pil kombinasi,
seperti:

Usia reproduksi
Telah memiliki anak ataupun belum
23

Gemuk atau kurus


Menginginkan metode kontrasepsi dengan efektivitas tinggi
Setelah melahirkan dan tidak menyusui
Setelah melahirkan 6 bulan yang tidak memberikan ASI eksklusif,
sedangkan semua cara kontrasepsi yang dianjurkan tidak cocok bagi
ibu tersebut
Pascakeguguran
Anemia karena berlebihan
Nyeri haid hebat
Siklus haid tidak teratur
Riwayat kehamilan ektopik
Kelainan payudara jinak
Kencing manis tanpa komplikasi pada ginjal, pembuluh darah, mata,
saraf
Penyakit tiroid, penyakit radang panggul, endometriosis, atau tumor
ovarium jinak
Menderita tuberculosis (kecuali yang sedang menggunakan rifampisin)
Varises vena

i)

Kontraindikasi

Mutlak
Adanya tumor yang dipengaruhi estrogen, seperti kanker
payudara
Penyakit hati yang aktif, baik akut maupun menahun
Pernah mengalami tromboflebitis, tromboemboli, kelainan
vaskular
DM
Kehamilan atau dicurigai hamil
Relatif
Depresi
Migrain
Mioma uteri
Hipertensi
Oligomenorea dan amenorea
j) Pemberian pil kombinasi pada wanita dengan keluhan di atas
perlu diawasi secara teratur dan terus-menerus, minimal sekali 3
bulan.

Kontraindikasi lainnya (yang tidak boleh menggunakan pil kombinasi)


24

Perdarahan pervaginam yang belum diketahui penyebabnya


Perokok dengan usia >35 tahun
Riwayat penyakit jantung, stroke, atau tekanan darah >180/110
mmHg
Riwayat gangguan factor pembekuan darah atau kencing manis
>20 tahun
Migraine atau gejala neurologic fokal (epilepsy/riwayat epilepsy)
Tidak dapat menggunakan pil secara teratur setiap hari
Pasien Anemia bulan sabit
k) Kelebihan

Efektivitas tinggi (95-98%), hampir menyerupai tubektomi bila


digunakan setiap hari
Risiko terhadap kesehatan sangat kecil
Frekuensi koitus tidak perlu diatur. Tidak mengganggu hubungan
seksual
Siklus haid menjadi teratur, darah haid berkurang (mencegah anemia)
Keluhan dismenorea primer berkurang atau hilang sama sekali
Dapat digunakan jangka panjang selama perempuan masih ingin
menggunakannya untuk mencegah kehamilan
Dapat digunakan sejak usia remaja hingga menopause
Mudah dihentikan setiap saat
Kesuburan segera kembali setelah penggunaan pil dihentikan
Dapat digunakan sebagai kontrasepsi darurat
Membantu mencegah: kehamilan ektopik; kanker ovarium; kanker
endometrium; kista ovarium; penyakit radang panggul; kelainan jinak
payudara

l)
67. Cara kerja AKDR
a) Mengentalkan cairan serviks
b) Merangsang kontraksi tuba fallopi
c) Menimbulkan peradangan kronis pada endometrium
d) Menimbulkan peradangan steril pada uterus
e)
f) Mekanisme Kerja akdr
25

Menimbulkan reaksi peradangan endometrium yang disertai dengan


sebukan leukosit yang dapat menghancurkan blastokista atau sperma
Menghambat kemampuan spermatozoa untuk masuk
ke dalam
saluran tuba
Mempengaruhi fertilisasi sebelum ovum mencapai kavum uteri
Mencegah sperma dan ovum bertemu
Memungkinkan untuk mencegah implantasi ovum ke uterus.
68. Kosong
69. Wanita 20 tahun, G1P0A0, usia kehamilan 2 bulan, bercak darah. Awal
periksa nyeri mules tidak ada. Setelah istirahat, kemudian muncul mules
dan nyeri seperti mau haid ditambah dengan keluarnya lendir. Apa yang
dilakukan selanjutnya ?
a) Tirah baring
d) Tidak bersenggama
b) Obat penguat kehamilan
e) Tidak melakukan kerja berat
c) Konsultasi ke bidan / ke
dokter
70. Kosong
71. Kosong
72. Seorang wanita 25 tahun, keputihan, gatal-gatal, hasil swab vagina
dengan parasit lonjong berflagel. Penyebab leukorea pasien diatas adalah

a) Trikomonas vaginalis
d) Bakteri
vaginosistest
b) Kandida albicans
amin KOH 10%
c) Klamidia trakomatis
73. Kosong
74. Bayi letak lintang teraba 2 kaki. Posisi apa?
a) incomplete
c) franc brach
b) complete
d) footling
e)
75. Wanita 24 tahun, umur kehamilan 37 minggu, pemeriksaan dalam
teraba dua kaki dan bokong dibagian bawah. Diagnosisnya
a) Incomplete
c) franc brach
b) complete
d) footling
76. Pada pembukaan serviks 4 cm, setelah berapa jam lagi dilakukan
evaluasi?
a) 2 jam
c) 6 jam
b) 4 jam
d) 8 jam
77. BB bayi 2.250 gram? BBLR
78. Batas BB lahir dikatakan normal apabila 2.500 gram - 3500
79. Kosong
80. Kosong
81. IUGR adalahpenyebab IUGR ibu sakit kronis ...
a) berada atau di bawah
c) berada di bawah persentil 5
persentil 5
d) berada
di
bawah
b) berada atau di bawah
persentil 10
persentil 10

26

e) berada di bawah persenti


15
82. Jika anak mengalami inkompatibilitas Rh terhadap ibunya, sebaiknya
kapan diberikan pengulangan IgD ?
a) 24 jam postpartum
d) 12 jam postpartum
b) 48 jam post partum
e) 7 hari postpartum
c) 72 jam postpartum
83. Kenapa bayi makrosomia pada DM gestational?
a) Adanya hambatan aliran glukosa fetoplasenta
b) Peningkatan jumlah dan densitas GLUT 1 pada sinsitiotrofoblast dan
sel endotel fetus
c) Peningkatan jumlah dan densitas GLUT 4 pada sinsitiotrofoblast dan
sel endotel fetus
d) Peningkatan difusi sederhana glukosa ke fetus
e) Unit fetoplasenta tidak memiliki mekanisme untuk mencegah
kelebihan glukosa yang ditransfer melalui plasenta
84. Pemenuhan kebutuhan karbohidrat pada fetus melalui hemeostasis ibu
dengan cara ?
a) Adanya hambatan aliran glukosa fetoplasenta
b) Peningkatan
jumlah
dan
densitas
GLUT
1
pada
sinsitiotrofoblast dan sel endotel fetus
c) Peningkatan jumlah dan densitas GLUT 4 pada sinsitiotrofoblast dan
sel endotel fetus
d) Peningkatan difusi sederhana glukosa ke fetus
e) Hemodilusi
85. Perubahan fisiologi pencernaan di masa kehamilan adalah
a) Penurunan pengosongan
c) Peningkatan
penyerapan
gastric-- estrogen
nutrisi di usus
b) Peningkatan motilitas usus
86. Kebutuhan kalsium dan fosfor tertinggi pada masa kehidupan
a) remaja
c) manula
b) kehamilan
87. Wanita PMS (pre-menstrual syndrome), terapi yang diberikan untuk
mengurangi keluhan pasien, kecuali ..
a) senam aerobik
d) diuretik tiazid
b) konsultasi psikis
e) calcium
c) SSRI
88. Pasangan suami istri belum punya anak, hasil analisa sperma normal,
HSG ditemukan adanya obstruksi parsial tuba, tindakan apa yang kita
lakukan ?
a) ulang analisa sperma
d) senggama terjadwal
b) laparotomi diagnostik
e) induksi ovulasi
c) post coital test
f)
g) UJI PASKA SENGGAMA
27

h) Sebenarnya belum ada kesepakatan tentang pelaksanaan uji in


meliputi : kapan dilakukan, berapa hari dibutuhkan abstinensi
sebelum pemeriksaan, kapan waktunya setelah senggama, dan
berapa banyak spermatozoa yang
harus tampak dalam 1
lapangan pandang besar/LPB.
i) Uji ini sangat berguna untuk menyelidiki adanya factor imunologi
apabila ternyata uji pascasenggama selalu negative atau kurang
baik, sedangkan kualitas air mani dan Tntibo serviks normal.
Perbandingan banyaknya spermatozoa yang gemetar di tempat,
yang maju pesat dan tidak bergerak mungkin menentukan
prognosis fertilitas pasangan itu.
j) Tahapan pemeriksaan dan penanganan
Infertilitas

28

k)

l)

m)

A.
PEMERIKSAAN INFERTILITAS DASAR

n) 1. Pendataan awal

o)

p)

q) a. Riwayat pasien, meliputi riwayat kesehatan,


perkawinan, hubungan suami-istri, kehamilan (bila
pernah), siklus haid, dan sebagainya.
t) b. Pemeriksaan fisik umum (tekanan darah, berat
badan, tinggi badan)
w) c. Pemeriksaan fisik ginekologis (kandungan).

r)

s)

u)

v)

x)

y)

z) d. Pemeriksaan fisik urologis/andrologis

aa)

ab)

ae)

af)

ah)

ai)

ak)

al)

an)

ao)

at)

av)

ac)
e. Pemeriksaan
transvaginal/transabdominal/
ad) transrektal
ag)
f. Pemeriksaan USG testis dan prostat

USG

aj) 2. Pemeriksaan laboratorium

am)

a. Pemeriksaan darah rutin dan urin

ap)
b. Pemeriksaan hormon reproduksi:
aq) FSH, LH pada hari ke 7-8 siklus haid.
ar)
Prolaktin, estradiol, progesteron, testosteron
pada hari ke 21-22 siklus haid.
as)
(dengan perkiraan siklus haid + 28 hari).

au)
aw)
ax)

ay)

c. Pemeriksaan infeksi TORSH-KM

29

az)
ba)
(jika

bb)

d. Analisis sperma

bc)

bd)

be)

e. Pemeriksaan Imunoandrologis

bf)

bg)

bh) 3. Biopsi endometrium (mikrokuretase) pada hari ke


21-22 siklus haid.
bk) 4. Hidrotubasi pada hari ke 9-10 siklus haid.

bi)

bj)

bl)

bm)

bn) 5. Histerosalpingografi (HSG) pada hari ke 9-10


siklus haid.

bo)

bp)
(jika

bq)
6. Histerosonografi pada hari ke 9-10
siklus haid

br)

bs)
(jika

bt)

bu)

bv)

bw)
B. PENANGANAN INFERTILITAS
DASAR

bx) by)
Sua Istri

30

bz)

Pemberian pengobatan

ca)

cb)

cd)

ce)

cg)

ch)

ci)

cj)

ck)

cl) C. PEMERIKSAAN INFERTILITAS LANJUTAN

cm) cn)
Sua Istri

cc)

Pemicuan ovulasi dengan pembuahan


alami

cf) Pemicuan ovulasi dengan inseminasi-intrauterin


buatan suami (IBS)

co)

1. Laparoskopi diagnostik/operatif

cp)

cq)

cs)

ct)

cu)

cv)

cw)

cx)
D. PENANGANAN INFERTILITAS
LANJUTAN

cy)
cz)
Sua

cr) 2. Biopsi testis

da)

Fertilisasi In Vitro (Bayi Tabung)

db)

dc)

dd)
de)
89. Wanita 19 tahun datang dengan keluhan haid tidak teratur 2 bulan
sekali, menyangkal pernah intercouse,diagnosis pasien di skenario ...
a) Anovulatorikelebihan
b) endometriosis
estrogen
31

90. Bayi normal, cukup bulan, selama hamil Ny.Cindy malas minum susu
dan malas beristrahat, BBL 2250 gram. Diagnosisnya
a) NBC- SMK
d) NCB Dismatur
b) NCB BMK
e) NCB malnutrisi
c) NCB KMK
91. Wanita intercouse usia 18 tahun, menarche 14 tahun, post coital
bleeding. Pemeriksaan apa yang dapat dilakukan?-->harusnya IVA
a) kadar TSH
c) rasio LH:FSH > 3
b) kadar CA 125
92. Seorang wanita mengeluh sejak 7 bulan yang lalu tidak memiliki gairah
seksual tetapi tetap bisa melayani suaminya. Gamgguan apakah itu?
a) disfungsi ereksi
c) gangguan
kenikmatan
b) gangguan hasrat seksual
seksual
d) gngguan orgasme
93. DM, gangguan fungsi seksual? ED
94. Kosong
95. Fase seksual dimana pada laki-laki tidak dapat refrakter sebelum
selesai siklus,sedangkan pada wanita langsung dapat refrakter adalah
Fase orgasme
96. Kosong
97. Tujuan umum kesehatan reproduksi remaja ...Semua benar
98. Pembangunan kesehatan merupakan investasi untuk peningkatan SDM
karena
a) Pembangunan kesehatan merupakan upaya seluruh potensi bangsa
Indonesia baik masyarakat,swasta, dan pemerintah.
b) Pengukuran IPM dan salah satu indikator selain pendidikan
dan ekonomi
c) Pelayanan kesehatan masyarakat pada daerah pedesaan
d) Masyarakat miskin bisa berobat gratis
e) Indonesia terkait kemitraan global
99. Paradigma baru pembangunan kesehatan?
a) Kuratif dengan alat yang
d) Peningkatan
akses
canggih
masyarakat
terhadap
b) Promosi dan pencegahan
kesehatan
penyakit
e) Peningkatan
mutu
c) Fokus pada wabah dan
pelayanan kesehatan
penyakit infeksi
100. Neonatal tetanus, case fatality rate. Pernyataan yang benar adalah
101.Indikasi penilaian kesehatan IPM
a) CFR, umur harapan hidup
d) UHH, infant mortality
b) insidensi,
rate,
life
rate, MMR
expectacy rate
e) MMR, IMR, life expectacy
c) prevalensi
rate,
life
rate
expectacy
rate,
mother
mortality rate
f)
32

g)
h)
i) SOAL UJIAN BLOK REPRODUKSI
j)
1. bentuk PAP seperti segitiga, diameter transversal terbesar terletak
diposterior dekat sacrum, dan dinding samping panggul membentuk
sudut yang makin sempit ke arah bawah. Tipe panggul menurut
klasifikasi Caldwell:
k) a. anthropoid
n) d. platypelloid
l) b. android
o) e. campuran
m) c. gynecoid
p)

Pelvis Gynecoid

Bentuk ini adalah yang khas bagi wanita

Diameter sagitalis posterior hanya sedikit lebih


pendek dari diameter sagitalis anterior

Batas samping segmen posterior membulat dan


segmen anterior juga membulat dan luas

Diameter transversal kira-kira sama panjangnya


dengan diameter antero posterior hingga bentuk p.a.p mendekati
bentuk lingkaran ( bulat)

Dinding samping panggul lurus, spina ischiadika


tidak menonjol, diameter inter spinalis 10 cm atau lebih

Incisura ischiadika mayor bulat

Sacrum sejajar dengan symphysis dengan


konkavitas yang normal

Arcus pubis luas

Pelvis android

Diameter sagitalis posterior jauh lebih pendek dari


diameter sagitalis anterior

Batas samping segmen posterior tidak membulat dan


membentuk sudut yang runcing dengan pinggir samping segmen
anterior

Segmen anterior sempit dan berbentuk segitiga

Dinding
samping
panggul
convergent,
spina
ischiadika menonjol,arcus pubis sempit

Incisura ischiadica sempit dan dalam

Sacrum letaknya ke depan hingga diameter antero


posterior sempit pada p.ap. maupun p.b.p

Bentuk sacrum lurus,kurang melengkung, sedangkan


ujungnya menonjol ke depan
q)
r)
Pelvis anthropoid
33

2.

3.

4.

Diameter antero posterior dari p.a.p lebih besar dari


diameter transversa hingga bentuk p.a.p lonjon ke depan

Bentuk segen anterior sempit dan runcing

Incisura ischiadika mayor luas

Dinding samping convergent,sacrum letaknya agak


ke belakang hingga ukuran antero posterior besar pada semua bidang
panggul

Sacrum
bisanya
mempunyai 6 ruas, hingga panggul
anthropoid lebih dalam dari panggulpanggul lain
Pelvic platypelloid

Bentuk ini sebetulnya


panggul genecoid yang picak,diameter
antero
posterior
kecil,diameter
transversal biasa

Segmen anterior lebar

Sacrum melengkug

Incisura ischiadika lebar


s)
wanita 19 thn, nyeri pinggul saat intercourse, memberat dalam 2 minggu.
Menarche 14 thn, riwayat menstruasi normal, obese, tidak ada
peningkatan suhhu. Kemungkinan hasil lab?
t)
a. tes kehamilan (+)
b. TSH
c. CA 125

u)
d. rasio LH:FSH >3
e. PL >3x normal+
v) kemungkinan dx.nya endometriosis krn ada nyeri pinggul yang
memberat.
Jarak terpendek antara promontorium sacrum dengan simfisis di sebut
a. diameter interspinarum
d. diameter obstetrik
b. konjugata vera
e. diameter biparietal
c. diameter diagonal
Distansia spinarum ; jarak antara linea anterior superior kiri dan
kanan
Konjugata sejati = Konjugata vera : jarak dari pinggir atas simfisis
ke promontorium
Konjugata diagonalis : jarak bagian bawah simfisis ke promontorium
= lebih dari 11,5 cm
Diameter anteroposterior bidang Inlet : jarak terdekat antara
promontorium dan simfisis = Konjugata obstetrika = Konjugata
diagonalis 1,5 sampai 2 cm
pada kala 3 ,perdarahan 100-200 cc,dengan plasenta tidak lengkap,uterus
berkontraksi minimal dan terjadi perdarahan kala 4 :
a. atonia uteri
b. prolaps uteri
34

c. ruptur uteri
d. trauma
5. Batas kista ovarii yang perlu di operasi
a. >4
c. >6
e. >10
b. >5
d. >7
f.
g.
h. 5. wanita 28 tahun, menstruasi lebih banyak dan panjang 1 tahun
terakhir makin memberat. pemeriksaan fisik: uterus membesar
setinggi simfisis pubis. PA : massa solid 5cm, batas tegasm warna outih
abu2m padat, eknyal.
i. a. rabdomyoma
l. d. lemyoma
j. b. lipoma
m. e. endometriosis
k. c. fibroma
6. SEmbelit pada kehamilan muda normal disebabkan :
a. kurangnya makan yang bias dimakan akibat rasa mual
b. penurunan peristaltic usus akibat peningkatan estrogen
c. penurunan peristaltic usus akibat peningkatan progesteron
d. penurunan peristaltic usus akibat peningkatan hCG
7. Apa penyebab keluarnya lendir??? PROGESTERON
n. Fisiologis dan patologis
o. Fisiologis : pada neonatus, saat ovulasi, saat dirangsang, terdapat
penyakit kronis, saat mendekati menarche.
p. Patologis : sebagian besar akibat infeksi bakteri, akibat massa pada
lumen sal. genital, keganasan. Biasanya terjadi perubahan pada
konsistensi, jumlah, warna, bau, dan substansi penyusun lendir.
8. Wanita, 20 tahun, GIP0A0, UK 32 minggu, KU perubahan penglihatan
dan pusing yang menetap dan tak menghilang dengan istirahat/obat
penghilang rasa sakit. Di RS TD 158/98 mmHg, kejang tonik klonik,
manajemen ?
a. stabilisasi
dengan
d. terminasi kehamilan dengan
diberikan MgSO4
induksi persalinan
b. stabilisasi dengan diberikan
e. terminasi kehamilan dengan
diazepam
SC
c. diberi obat anihipertensi
9. Ibu HBSAg (+), kapan anaknya dapat HBIg? 12 jam pertama (2x24
jam setelah lahir, 12 jam, 1 mg, 6 bl)
10. Ny Ita, 25 tahun, didiagnosa menderita herpes genitalis. Obat pilihan
utama untuk penyakit ini adalah Acyclovir. Saat ini Ny Ita sedang
menyusui (ASI ekslusif) bayinya yang berusia 3 bulan. Bagaimana
pemberian terapinya ?
a. tetap diberikan Acyclovir bisa jg valacyclovir
b. diganti dengan Famcyclovir dihindari saat laktasi
c. Pemberian antiviral oral ditunda dan cukup diberi antiviral topical
d. Pemberian antiviral oral ditunda dan cukup diberikan antipruritus dan
analgesic
e. dapat diganti dengan Itraconazole tidak direkomendasikan saat
laktasi
35

11. Acyclovir termasuk golongan apa pada derajat FDA untuk ibu hamil
12.
a. golongan B
b. Golongan C
c. Golongan
D
d. Golongan X
13. Ami, 27 tahun, hamil 24 minggu. Seminggu gatal-gatal, GO, terapi
cipro 2x500 mg 5 hari. Langkah yang harus diambil dokter :
a. tetap memberi cipro
d. nistatin supp.--> antifungal
b. hanya diberi histamine
utk candida
c. diganti dengan cefadroxil
2x500 mg
14.
Ket:
Ciprofloxasin
termasuk
golongan
floroquinolon,
tidak
direkomendasikan pada ibu hamil, kategori C pada kehamilan,
menyebabkan kerusakan kartilago pada hewan coba. Bisa
diekskresikan melalui ASI, tp menurut AAP masih dapat diberikan
pada laktasi.
Neisseria gonorrhoeae termasuk bakteri diplokokus aerob gram
negatif.
Cefadroxil (beta laktam antibiotik) termasuk golongan cephalosporin
generasi pertama, pemberian per oral, antibiotik broad spektrum
untuk bakteri gram negatif dan positif, terutama untuk bakteri cocci
gram positif. Aktifitas lebih rendah pada bakteri anaerob.
15. Disekresikan dalam jumlah rendah pada Asi, dan masih
kompatibel untuk laktasi.
16. wanita, 32thn datang dengan nyeri perut dan perdarahan pervaginam.
Mens terakhir 8 minggu lalu. Tes kehamilan (+). Cairan bebas di cavum
uteri (NB. mungkin maksudnya cavum douglas) dan iritasi peritoneum.
Apa diagnosis yang tepat? KET
17.
MK KET :
18.
TFU lebih kecil dari usia kehamilan, Nyeri perut bawah, amenore,
perdarahan pervaginam, nyeri goyang portio, ada massa di adneksa.
19. Wanita, 21 tahun, hamil 37 minggu, perdarahan, tidak nyeri, gerak
anak masih ada, KU : baik, TD 110/70, N 80x, Leopold teraba masaa
panggul, Djj baik, his (-), DD :
a. tumor
jalan
lahir
(gak
c. plasenta previa
perdarahan)
d. kehamilan abdominal
b. suspect
solusio
plasenta
e. gangguan bekuan darah
(kondisi janin tdk baik)
20. 24 tahun, hamil ke 4, Abortus 3, HTA 20-05-09. Untuk menjaga
kehamilan, progesterone diberi hingga usia :
a. 5 minggu
d. 17 minggu
b. 9 minggu
e. 21 minggu
c. 13 minggu
f.Jika
aborsi
disebabkan
oleh
defisiensi
progesteron
akibat
pengangkatan korpus luteum, umumnya terjadi sangat cepat pd UK 4-7
mgg (terapi dilakukan pd mgg2 ini), terapi pengganti dapat diberikan
36

sampai usia kehamilan 8 10 minggu (dimana plasenta sudah dpt


mensekresi progesteron).
g.
Pada abortion habitualis, terapi hormon umumnya tidak perlu
kecuali pd kasus gangguan fungsi tiroid atau gangguan fase luteal.
21. Gejala utama hymen imperforate :
a. tidak pernah haid
d. benjolan
b. nyeri perut
e. PMS
c. dismenorea
22. Terapi terhadap hymen imperforate
a. marsupinlisasi
(kista
kel.
c. eksisi
Bartolin)
d. ekstirpasi
b. insisi
e. pungsi
23. pemeriksaan gula darah dalam kehamilan di lakukan pada umur
kehamilan ?
a. 12 mgg
d. 8 mgg
b. 36 mgg
e. 32 mgg
c. 24 mgg
f.Untuk
skrining
terhadap
GMD,
pemeriksaan
gula
darah
direkomendasikan pada usia kehamilan 24 28 mgg.
24. ny. E 48 th. Ku: keluar darah banyak dan nyeri perut bawah, RPD : haid
tidak teratur di sertai ada gangguan di bawah perut, PF : KU: lemah dan
pucat,abdomen, ada masa 10X10 cm. Px.dalam : ada masa
kenyal,kehamilan positif: 10 minggu. Saat perut di goyangkan masa ikut
bergerak. berdasarkan PF dan keluhan maka diagnosis?
a. kistaria ovari
c. mioma uteri
b. KET
d. kahamilan 10 minggu
25. Pemeriksaan untuk diagnose pasti KET :
a. USG
c. RT
d. Foto
b. CTG
rontgen
26. wanita 20 tahun,G1PoAo, kehamilan 38 minggu,kontraksi reguler 3-4
menit( 60 detik), penipisan serviks 90 %,bukaan 3 cm. 1 jam kemudian
penipisan serviks 100%, dan bukaan 5 cm.
a. pecahkan ketuban
c. apapun tidak di lakukan
b. augmentasi dengan oksitosin
d. SC
drip
e. anastesi epidural
f.
Masuk fase aktif Kala I, lamanya < 7 jam. Tunggu sampai bukaan
lengkap, 1cm/1,5jam pd nullipara, dan 1cm/1,2 jam pd multipara.
27. Pasien 20 thn, G1P0A0. Usia kehamilan 38 minggu datang ke klinik
dengan kontraksi yang regular setiap 3-4x/10mnt masing2
selama 60 dtk. Pemeriksaan dalam menunjukkan penipisan serviks
90 % dan pembukaan 3cm. Satu jam kemudian penipisan
menjadi 100% dan pembukaan 5 cm. Pasien diatas berada pada
tahap
a. periode akselerasi fase laten kala I
b. periode deselerasi fase laten kala I
c. periode akselerasi fase aktif kala I
37

d. periode deselerasi fase aktif kala I


e. periode maksimum loope fase aktif kala I
f.
g.
Fase-fase dalam fase aktif kala I
a. Fase akselerasi: dalam 2 jam pembukaan mencapai 4cm.
b. Fase dilatasi maksimal: dalam waktu 2 jam pembukaan berlangsung
sangat cepat, dari 4 cm menjadi 9 cm.
c. Fase deselerasi: pembukaan menjadi lambat kembali. Dalam waktu 2
jam pembukaan dari 9 cm menjadi lengkap.
h.
28. Wanita, 2 hari postpartum, tidak menyusui anaknya, datang dengan
keluhan payudara nyeri dan bengkak. Tatalaksana yang sesuai:
a. dikompres dan dibebat
c. berikan bromokriptin
d. Analgesik
b. ASI diperas
i. Ny. S melahirkan bayi ke-6 dengan berat 2,25 kg. Selama 9
bulan kehamilan dia malas minum susu dan kurang bisa
istirahat. Ny.S merasa bersalah karena berat anaknya tidak
sebesar berat anak-anaknya sebelumnya.
29. Diagnosis yang tepat
a. Bayi prematur
d. BBL ringan
b. Posterm
e. BBL kecil
c. BBLR
30. Ny Cindy G6P6A0 BB bayi 2250 gr, lebih rendah dari yang lain, bayi
normal, cukup bulan, Bayi malas minum susu dan sulit istirahat,
diagnose dr.Joko :
a. NCB-SMK (prematur)
d. NCB-dismatur
b. NCB-BMK
(heavy
e. NCB-malnutrisi
weight)
f.
c. NCB-KMK
g. Nb: Dismatur dan Malnutrisis merupakan penyebab KMK (kecil
menurut usia kehamilan)
h.
31. PAP dikatakan sempit bila : Konjugata obstetri < 10 cm , diameter
transversum terbesar < 12 cm dan konjugata diagonal < 11,5
cm
32. Perubahan system respirasi selama masa puerperium :
a. penurunan
volume
c. lupa
tidal
d. lupa
b. penurunan
volume
total
e.
Pada masa kehamilan perubahan fisiologi pernapasan yg terjadi
antara lain :kapasitas vital (100-200 ml),kapasitas inspirasi (300 ml),
volume cadangan ekpirasi (dari 1300ml jadi 1100ml), volume
residual (dari 1500ml jadi 1200ml), functional residual capacity
38

(500ml), volume tidal (dari 500ml jadi 700ml), minute ventilation


(40%) krn terjadi volume tidal akibat tdk terjadi perubahan RR.
33. Hal yang membedakan antara psikosis post partum dengan psikosis
biasa adalah :
a. gejala
c. terapi
b. penatalaksanaan
d. onset keluhan
e.
Psikosis postpartum bisa terjadi cepat dalam 42-72 jam
pospartum dan namun biasanya berkembang dalam 2-3 minggu PP
dengan gejala yg dramatis dan mendadak.
34. Menurut pedoman PPDGJ III, gangguan pada masa nifas menurut onset
: Dalam 6 minggu PP
f.Beberapa gangguannya: depresi pospartum (gangguan jiwa ringan),
psikosis postpartum (gangguan jiwa berat), dan baby blues
g.
Wanita 32 tahun. G1P0A0 datang kerumah sakit jam 6. ......
3jam lalu disertai lendir darah. TFU teraba 2 jari dibawah presesus
xyphoideus. Puka, preskep, sudah masuk PAP, kesan panggul normal.
bukaan 2 dengan HIS 284 detik/mnt, 12 jam kemudian bukaan 6 dengan
HIS 3*5 detik/mnt
h.
Analisis His adekuat dan semakin meningkat, tp dilatasi tg
terjadi lambat. Ini merupakan tanda dr incoordinated uterine contraction
sehingga tdk mampu menimbulkan dilatasi serviks.
35. Dari skenario diatas kemungkinan pasien mengalami:
a. Coordinate uterine contraction His kuat & sinkron, persalinan
cepat
b. Hypotonic uterine contraction His lemah primer/sekunder, fase
laten memanjang/fase aktif memanjang & secondary arrest.
c. Incoordinated uterine contraction
d. Mixed uterine contraction
e. tidak ada jawaban yang benar
36. penyebab terjadinya kelainan di atas adalah:
a. stress
d. genetik
b. primi
e. semua benar
c. usia 30 tahun
f.
37. Bila selama observasi, DJJ 180x/menit, tindakan yang tepat :
a. augmentasi
d. SC
b. induksi
e. ditunggu sampai terjadi
c. ekstraksi forceps
kelahiran pervaginam
f. Jika terjadi pd kala I SC, jika pd Kala II ekstraksi
vakum/forceps
38. Haid 2 bulan sekali, sekalinya haid selama 4 hari, sampai ganti
pembalut 3x. Riwayat menarche 15 tahun
a. setiap bulan
d. terapi hormonal
b. setiap 3 bulan
e. terapi diagnostik
c. terapi konservatif
39. wanita 26 tahun. Amenorhea sekunder sejak 4 bulan lalu. Didapat hasil
labolatorium peningkatan prolaktin (100 ng/ml, normalnya <25ng/ml
39

pada wanita tidak hamil) dan hCG (+). Manajemen selanjutnya yang
tepat pada kasus adalah: Dx. Hamil
a. CT scan sella tursika untuk mencari adenoma pituitari
b. evaluasi kemungkinan hipotiroidisme
c. evaluasi ulang prolaktin serum, pastikan kadarnya tidak lebih
dari 300 ng/ml
d. permberian bromokriptin untuk supresi prolaktin utk adenoma
hipofisis
e. perawatan obstetrik rutin
f.pada kehamilan kadar prolaktin normalnya meningkat 150 ng/dl,
puncaknya kadar prolaktin mencapai 10.000 ng/dl pada minggu ke 20
26, dan selanjutnya menurun dan mencapai level terendah pada
minggu ke 34 kehamilan.
40. wanita 39 tahun, anak 3. Dismenorea berat dan menorhagia.
Uterus teraba besar, difus, tegang, nyeri sekitar adneksa
Adenomiosis
g.
Endometriosis nyeri dengan pola siklik termasuk dismenore
sekunder, nyeri menjalar sampai ke rektum, daerah sakrum bawah dan
makin lama makin hebat, pd pemeriksaan dalam teraba massa dgn
nyeri tekan, adneksa membesar lunak dan nyeri, 75% infertil.
41. wanita 19 tahun, 46 kg, 153cm, telat haid 3 bulan: mekanisme
telatnya?
a. Stressinaktivasi
c. inhibisi CRH jika jd
amigdala
amenore
b. menurunnya
kortisol
d. stimulasi
pelepasan
darah
endorfin dan ACTH
e. hambat FSH
42. hormon yang di lepaskan hipotalmus saat stress adalah :
a. cortisol
d. CRH
b. TRH
e. endorphin
c. ACTH
f.
g. 65. lakuna kecil sel granulosum berisi cairan
h. a. folikel sekunder
k. d. folikel de graf
i. b. folikel primer
l. e. folikel primordial
j. c. folikel tersier
m.
43. Keterlibatan neurotransmitter adalah salah satu etiologi PMS,
yatu :
a. dopamine
c. GABA
d. NE
b. serotonin ( sekresi)
e.
f. 68. berapa batas diameter kista ovarii, baru indikasi operasi:
g.
a. >4
b. >5
c. >6
d. >7
e. >10
h.
40

44. Hasil pemeriksaan pap smear wanita 35 tahun G3P5 menunjukkan


high grade squamous intraepithelial lesion of the cervix (CIN
III). Pada cone biopsy menunjukkan squamous cell carcinoma
dengan invasi 1 mmdi bawah membrane basalis, stage ?
a. carcinoma of low malignant potential
b. microinvasive cancer, stage 1a1
c. atypical squamous cell of undetermined significance (bkn
karsinoma)
d. Ca in situ (bm invasi)
e. invasive cancer, stage 1a1
f.
45. Seorang wanita 20 tahun datang ke poli ginek dengan keluhan ada
benjolan pada introitus vagina. Benjolan itu sudah ada sejak
beberapa bulan yang lalu dan dirasa semakin membesar. Suaminya
juga memiliki benjolan yang sama pada penisnya. Pada
pemeriksaan fisik didapatkan lesi multiple dengan ukuran 2-10 mm
pada introitus vagina disertai kondiloma. Tak ditemukan lesi pada
serviks. Dari pemeriksaan pap smear didapatkan gambaran atypical
squamous cells of undetermined (ASCUS) dengan jenis HPV tak
beresiko tinggi. Tipe virus manakah yang memiliki kaitan erat dengan
kondiloma pasien ? tidak beresiko tinggi type 6 dan 11
a. tipe 11
d. tipe 45
b. tipe 16
e. tipe 56
c. tipe 18
46. HPV tipe: 11, 16 atau 18?????
47. wanita, 25 tahun. Sudah menikah tapi belum punya anak. Gejala
keputihan dengan warna kekuningan, bau dan gatal sekali.
Ditemukan gambaran mikroskopik parasit lonjong dengan flagella.
Penyebab leukorhea adalah:
a. Trichomonas vaginalis (protozoa bergerak dgn 4 flagel
dan nukleus di tengah)
b. Chlamydia frachimotis (bakteri intraseluler obligat dlm epitel
kolumnar); MK: duh purulan/mukoid, PCB, byk yg asimptomatis.
c. Candida albicans (hifa bercabang dan bertunas); MK: pruritus
berat, eritema, dysuria, duh putih kental berbau aneh, terlihat
spt keju lembut.
d. Naisseria gonorrhea (bakteri diplococus, gram negatif,
berkembang biak di dalam sel sekretori, pd sekret ada di dlm
WBC dan sel epitel); MK: vaginal discharge putih/kuning byk, tdk
bau, tdk iritasi, sering asimptomatis.
e. Bacterial vaginasis (clue cells vagina, tanpa WBC, cluster bakteri,
tes whiff KOH +) MK: sekret tipis putih non pruritik, bau anyir.
48. Wanita, 39 tahun, anakke 3, berencana untuk memakai IUD,
mekanisme kerja IUD ? Mekanisme peradangan lokal yg steril
sehingga menggagalkan implantasi
41

49. 22 tahun, konsul KB, rokok (-), alcohol (-), vital sign & pemeriksaan fisik
dbn. Mekanisme pil kombinasi :
a. menghambat langsung maturasi oosit
b. menghambat ovulasi
c. produksi secret uterus yang toksik terhadap embrio yang sedang
berkembang
d. perubahan hiperplastik endometrium akan menghambat
implantasi
e. obstruksi uterotuba yang akan menghambat sperma
50.
84. teknik tubektomi dengan pengikatan tuba menggunakan
bahan yang diarbsorpsi kemudian dipotong adalah??? Apa ayok????
pomeroy
51. Rujukan gawat darurat dalam Safe Motherhood, di urutan :
a. 1 KB
b. 2 ANC
c. 3 persalinan bersih dan aman
d. 4 pelayanan obstetrik essensial (termasuk penanganan
kegawatdaruratan)
e. 5
52. AKI di Indonesia masih tinggi, MPS ialah upaya untuk menurunkan.
Pesan MPS ialah kecuali : bank darah
53.
3 Pesan MPS
54.
(1) setiap persalinan ditolong oleh tenaga kesehatan terlatih,
55.
(2) setiap komplikasi obstetrik dan neonatal mendapat
penanganan adekuat, dan
56.
(3) setiap perempuan usia subur mempunyai akses terhadap
pencegahan kehamilan yang tidak diinginkan dan penanganan
komplikasi keguguran.
57.
4 strategi utama MPS
58.
(1) meningkatkan akses dan cakupan pelayanan kesehatan ibu
dan bayi baru lahir yang berkualitas, (2) membangun kemitraan yang
efektif melalui kerja sama lintas program, lintas sektor dan mitra
lainnya, (3) mendorong pemberdayaan perempuan dan juga keluarga
melalui peningkatan pengetahuan, (4) mendorong keterlibatan
masyarakat dalam menjamin penyediaan dan pemanfaatan pelayanan
kesehtan ibu dan bayi baru lahir.
59. Fluor albus : putih abu-abu; pH 5,5; ada clue cells; bau amis; gatalpanas-kental (-)
a. candidiasis
d. tak terbaca
b. bakterial vaginosis
e. Klamidia
c. trichomonas vaginalis
60. berdasarkan etiologi pada kasus. Faktor resiko apa saja yang infertilitas
a. faktor
pria
dan
c. idiopatik
kelainan tuba
d. kelainan uterus
b. endometriosis
61. Pemeriksaan untuk infertilitas pria :
42

a. analisa sperma
62. pilihan obat untuk menyebabkan kontraksi:
a. oksitosin (kontraksi)
c. Prostaglandin
b. alkaloid
ergot
(kontraksi)
(kontraksi)
d. B-2 reseptor antagonis
(relaksasi/tokolitik)
63. uterotonika yang baik digunakan untuk induksi persallinan tapi belum
ada pembukaan
a. Misoprostol pervaginam (prostaglandin)
b. Oxytosin drip (jika servix sudah matang sebelum induksi)
64. kapan plasenta matang??? Minggu 16
65.
66.
Efek samping oksitosin drip :
a. Rupture uteri
c. KPD
b. Plasenta previa
d. atonia uteri
67. wanita postpartum 5 bulan.. menysui bayinya ekslusif, belum juga
menstruasi sampe sekrang..
a. penurunan prolaktin
e.
b. penurunan beta hcg
f.
c. peningkatan oksitosin
g.
d.
h.

43

i. SOAL TAMBAHAN
j.
1. Jenis panggul yang oval, diamter anterior lebih besar dari diameter
transversal antropoid.
k. Pembahasan :
Lonjong di bawah
platipelloid
l.
Oval di atas
antropoid
m.
n.
Jantung android
o.
Bulat
ginekoid
p.
2. Wanita 15 thn, menarche umur 14 thn, nulligravida, Menyangkal
intercourse, gangguan haid, nyeri sampai pingsan, jarak lebih lebar, 12 hari tidak masuk sekolah anovulasi
q. Pembahasan : kapan endometriosis terjadi???
- Endometriosis wanita usia 25-35 thn, tinggi kurus, BMI rendah,
pernah dilaporin di anak usia 11 tahun
- Patofisiologi : Aliran balik darah haid yg mengandung sel-sel
endometrium yg masih hidup implan di tuba atau peritoneum.
Saat operasi terjadi implantasi.
- Tatalaksana tinggi progesteron, rendah estrogen, tinggi
testosteron
- Adenomeiosis terjadi pada orang tua
3. Cardinal movement
- Engagement : penurunan diameter terbesar kepala (biparietal)
- Descent : turun, penurunan bagian terbawah janin
- Flexion : kepala menekuk, dagu menyentuh ke dada
- Internal rotation : putaran kepala ke arah depan ke bawah simfisis
pubis, perubahan diameter biparietal menjadi diameter
suboksipitobregmatikus
- Extension : kepala mengalami ekstensi sehingga dapat melewati
vulvas door
- Eksternal rotation : putar kembali ke arah badan bayi
- Ekspulsion : lahirnya bahu depan diikuti dengan lahirnya bahu
belakang
4. Kapan wanita yang abortus mola hidatidosa bisa hamil lagi 1 tahun.
Observasi kemungkinan PTG dan karena kehamilan bisa mengganggu
hasil tes dalam follow up
5. Hormon yg dilepaskan hipofisis anterior FSH
6. Proteinuria +4, edema, TD 170/110 mmHg PEB
- TD > 160/110 mmHg
- Proteinuria > +1
- Edema paru nafas cepat ronkhi

- Nyeri epigastrium
- Gangguan visus
7. Obat antihipertensinya CCB (nifedipin)
- Jika DBP > 110 tidak boleh diberikan ACEI dan ARB pd kehamilan.
Diuretik : kalo dekomp kordis
8. Letak Janin :
- Preskep : UUK, UUB, dahi, muka
- Letsu : Frank, komplit, inkomplit, footling----breech
- Letli : ndak ada di fuundus dan di simfiisis
9. Frank breech (presbo) cuma dpt pantat
10.
Complete breech (presbokaki) pantat dan bagian kecil janin
kaki 2 buah
11.
Incomplete (bokong dan salah satu kaki)
12.
Footling : teraba 2 kaki tokkkkk
13.
Kelainan presentasi :
- Presentasi muka. Dari pemeriksaan dalam teraba : dagu, mulut,
hidung, orbita, dan adanya kaput suksadeneum
- Presentasi dahi : sutura frontalis, UUB, pangkal hidung dan lingkar
orbita
14.
Bayi presentasi belakang kepala (verteks), diamter yg masuk ke
PBP suboksipito bregmatikus
15.
Presentasi muka dagu posterior, tindakan SC
16.
Menghadap kaki Leopold IV
17.
Pemeriksaan PA dgn adneksa kulit di bawahnya Kista dermoid
18.
Kapan dikatakan IUGR/PJT < 10 persentil
19.
Merokok 25 batang faktor resiko apa?? apa-apa dah (no
idea)
20.
Peningkatan berat badan smpai hamil aterm 11-15 kg (N :
12,5)
21.
Kapan Ig D diberikan pd wanita dgn inkompatibilitas 28 dan 34
minggu
22.
Fase deselerasi pada kala I adalah 9-10 cm (fase maksimum
slope 4-9 cm), fase akselerasi 3-4 cm
23.
Tanda-tanda Cinta beserta hasilnya :
- Tanda Chadwick Livide vulva, serviks, vagina
- Tanda Hegar ismus uteri melunak, +panjang, hipertrofi
- Tanda Pischacek uterus menonjol pada salah 1 sisi tempat
implantasi
- Tanda Goodell servik menjadi lunak dan hipervaskularisasi
24.
Pemeriksaan Usia kehamilan dgn USG < 9 minggu
r. Tingkat kesalahan pemeriksaan Usg pada usia kehamilan :
s. 7-12 minggu : + 5 hari
t. 13-20 minggu : + 7 hari
u. 21-30 : + 14 hari
v. >30 : + 21 hari

25.
Nyeri perut bawah, perdarahan haid tidak teratur, pada
pemeriksaan dalam didapatkan masa tidak ikut bergerak kista
endometriosis
26.
Pemeriksaan penunjang USG
27.
Tindakan hormonal progesteron
28.
Dia hub sex pd hari ke 11 dari hari haid mekanisme kerjanya
adalah dgn menurunkan sekresi LH
29.
Obatnya karsinoma endometriosis adalah MDA (medroksi
progesteron asetat)
30.
Hormon yg menyebabkan pembengkakan pada payudara
estrogen, progesteron, somatomammotropin
31.
HPHT
w.
x.
y. SOAL UAS OBGYN II
z.
aa.
ab.
ac.

1. Gejala yang sering terjadi pada mioma uteri :


a. Hipermenorea
b. Dismenore

ad.

c. Retensio Urine

ae.

d. Obstipasi

af. 2. Leomiosarkoma terbanyak berasal dari :


ag.

a. Mioma submukosum

ai.

c. Mioma subserosum

ah.

b. Mioma intramural

aj.

d. Bukan salah satu diatas

ak.
al.

3. Mymoma geburt :

a.
Berasal
subserosum

dari

mioma

am.
b. Berasal dari mioma
intramural
ap.

an.
c.Tumor
berkonsistensi
berwarna putih

kenyal

ao.
d. Tumor berkonsistensi
padat berwarna hitam

4. DD mioma uteri, kecuali :

aq.
a.
Kehamilan
(dengan mioma subserosum)

as.

c.Adenomiosis
mioma subserosum)

(dengan

ar.

at.

d.Khoriokarsinoma
mioma subserosum)

(dengan

b.Inversio
uteri
mioma submukosum)

au.

(dengan

5. Perubahan sekunder mioma uteri, kecuali :

av.

a. Atrofi

aw.

b. Degenerasi hialin

ax.

c. Degenerasi kistik

ay.

d. Degenerasi putih

az.6. Komplikasi yang terjadi pada mioma uteri, kecuali :


ba.

a. Degenerasi ganas

bc.

c. Torsi

bb.

b. Leiomioma sarkoma

bd.

d. Koriokarsinoma

be.

7. Salah satu pengaruh mioma uteri dalam kehamilan :

bf.

a. Retensi urine

bg.

bh.

b. Abortus

bi.

c. Konstipasi
d. Hipertensi

bj. 8. Pengobatan mioma uteri, kecuali :


bk.

a. Harus dioperasi

bm.

c. Radioterapi

bn.

d. Miomektomi

bl.

b. CnRH

bo.

9. Komplikasi tumor ovarium, kecuali :

bp.

a. Perdarahan

br.

c. Putaran tungkai

bq.

b. Sindrom Coshirs

bs.

d. Sindrom Meigs

bt. 10. Akibat pertumbuhan tumor, kecuali :


bu.
bv.

a. Benjolan perut
b. Gangguan miksi

bw.

c. Diare

bx.

d. Obstipasi

by.11. Putaran tungkai pada tumor ovarium dapat menyebabkan :


bz.
a.
hemoragik

Nekrosis

cb.
cc.

c. Stroke hemoragik
d. Eklamsia

ca.

b. Sindrom turner

cd.

12. Perlekatan robekan dinding kista dengan dinding perut, disebut :

ce.

a. Limfangioma

cf.

b. Calcireus degeneration

cg.
c.
peritonei
ch.

Pseudomiksoma

d. Carneus

ci. 13. Sindroma meigs terdiri dari :


cj.

a. Kista dermoid

cl.

ck.

b. Asites

cm.

c. Hidrotoraks
d. B & C benar

cn.

14.Bila terdapat keganasan maka penatalaksanaannya adalah :

co.

a. Histerektomi

cp.

b. Salpingotomi

cq.
cr.

c. SOB
d. A & B benar

cs. 15. Yang termasuk sex cord stroma tumor adalah :


ct.

a. Androblastoma

cu.

b. Teratoma

cv.

c. Brenner

cw.

d. Korio karsinoma

cx.16. Berikut ini tumor jinak kistik :


cy.

a. Angioma

da.

c. Kista dermoid

cz.

b. Maskulinovo blastoma

db.

d. Teratoma

dc.

17. Faktor resiko mayor untuk terjadinya Ca servik :

dd.

a. HSV 2

df.

de.

b. HPV 31 33

dg.

dh.
di.

c. HPV 16 18
d. Klamidia

18. Prinsip kemoterapi Ca servik :

a. Bleocyn base
dj. b. Platanin base

dk.
dl.

c. Oncovin base
d. Mytomicin basePTG

dm. 19. Radikal terhadap Histelektomi dapat dilakukan pada kasus yang
masih menginginkan kehamilan pada stadium :
dn.
do.

Modified1a2-1b

dp.

a. III B

dr.

c. II A

dq.

b. II B

ds.

d. I A

dt. 20. Ekstensi paramedikal Ca servik dengan CFS 0%/10% berada dalam
stadium klinik :
du.
dv.

a. I A
b. II B

dw.

c. III B

dx.

d. I A

dy.21. Biopsi terarah pada lesi yang tidak kasat mata dilakukan dengan
tuntunan :
dz.

a. USG

ea.

b. Kolposkopi

eb.

c. Gastroskopi

ec.

d. Histeroskopi

ed. 22. Metastase jauh Ca servik ditemukan di paru2, maka stadium klinik
kasus diatas :
ee.
ef.

a. III A
b. III B

eg.

c. IV A

eh.

d. IV B

ei. 23. Skrining massa untuk deteksi dini Ca servik meliputi :


ej.

a. Hybride Capture II

ek.
en.

el.

b. PCR

c. Pap smear

em.

d. IV A

24. Spesimen untuk pemeriksaan pap smear harus mencakup :

eo.

a. Exfoliating Cell

eq.

ep.

b. Parabasal cell

er.

c. Basal cell
d. Intermediate cell

es.25. Ibu hamil dengan HIV perlu diberikan ART. Rekomendasi saat
pemberian adalah :
et.
a.
HIV (+)

Sejak

diketahui

eu.
b. Sejak usia kehamilan
> 13 minggu

ev.

c. Sejak usia kehamilan > 16


minggu

ew.
d. Sejak usia kehamilan
> 20 minggu

ex.

26. Transmisi vertikal HIV paling besar kemungkinan terjadi pada saat :

ey.

a. Antenatal

ez.

b. Intra partum

fa.

c. Post partum

fb.

d. Laktasi

fc. 27. Pencegahan transmisi perinatal dilakukan dengan :


fd.

a. Menurunkan maternal viral


load

ff.

fe.

b.
Menurunkan
maternal total

fg.
d. Semua
diatas benar

tranfusi

c. Menurunkan paparan janin


terhadap sekret ibu
jawaban

fh. 28. Yang tidak termasuk tindakan penanganan syok hemoragik :


fi.

a. Resusitasi cairan koloid

fj.

b. Infus cairan cristaloid

fk.

c. Baring terlentang dengan


kaki naik

fl.

d. Berikan O2 5-10 liter/menit


melalui kanula hidung atau
sungkup muka

fm. 29. Petunjuk adanya perbaikan dalam penanganan kusus dengan syok
hemoragik sbb, kecuali:
fn.
a.
Tekanan
darah
sistolik meningkat 90 mmHg

fp.

c. Produksi urine > 100 ml/4


jam

fo.

fq.

d. Pernapasan
saturasi normal

b. Kesadaran membaik

wajar

dan

fr. 30. Syok yang diakibatkn oleh berkurangnya tonus vaskuler akibat infeksi
disebut sebagai syok :
fs.

a. Septik

fu.

c. Hipovolemik

ft.

b. Neurogenik

fv.

d. Kardiogenik

fw. 31. Pada syok hemoragik tindakan utama yang harus dikerjakan dengan
urutan sbb:
fx.

a. Stabilisasi, perbaiki sirkulasi dan oksigenasi, penanganan


penyebab

fy.

b. Perbaiki sirkulasi dan oksigenasi, stabilisasi, penanganan penyebab

fz.

c. Penanganan penyebab, stabilisasi dan perbaikan sirkulasi

ga.

d. Oksigenasi, pemasangan infus dan menghentikan perdarahan

gb.
32. Pada late reversible shock stadium lanjut ditemukan
gambaran klinik sbb, kecuali:
gc.

a. Tekanan darah terus


menurun

gd.
b. Nadi makin cepat
dan kecil
gg.

ge.
c.
menurun
gf.

Suhu

badan

d. Tanda gangguan fungsi


alat2 vital mule nampak

33. Yang merupakan ciri histopatologi mola hidatidosa adalah:

gh.
a. Stroma villi khorialis
hipervaskuler

gj.

gi.

gk.

b. Stroma villi khorialis


avaskuler

c. Stroma
miometrium

villi

invasi

ke

d. Stroma villi mengecil

gl. 34. Salah satu gejala klinik yang khas pada mola hidatidosa adalah :
gm.
a.
geravidarum

Hiperemis

gn.
b.
Pembesaran
uterus lebih besar dari pada
yang seharusnya

go.
c.
pervaginam
gq.

Perdarahan

gp.

d. hCG (+)

35. Mola komplit terjadi karena :

gr.

a. Sperma membuahi
telur yang normal

sel

gs.

b. Sel telur dibuahi oleh 2 sel


sperma

gt.

c. Sperma membuahi
telur yang tidak berinti

sel

gu.
d.
2
sperma
membuahi 1 sel telur berinti

gv.36. 90% mola komplit memiliki kromosom :


gw.

a. 46 XY

gy.

gx.

b. 69 XXY

gz.

ha.

c. 69 XYY
d. 46 XX

37. Penatalaksanaan mola hidatidosa secara umum :

hb.

a. Histerektomi

hc.

b. Kemoterapi

hd.
c. Evakuasi dengan
kuret hisap
he.

d. Radioterapi

hf. 38. Yang penting pasca evakuasi mola hidatidosa, adalah :


hg.
a.
teratur
hh.
b.
kembali

Follow

up

yang

hi.
hj.

Segera

hamil

c. Kuret hisap ulang


d. Makan
bergizi tinggi

makanan

yang

hk. 39. Sebagian besar penyakit trofoblas ganas pasca mola hidatidosa,
adalah :
hl.

a. Mola parsial

hm.

b. Mola destruens

hp.

c. Koriokarsinoma

ho.

d. Embrional karsinoma

40. Dasar diagnosis PTG, adalah :

hq.
a.
Titer
menetap/meningkat
hr.

hn.

hCG

b. Perdarahan pervaginam
yang berulang (pasca evakuasi)

hs.

c. Tanda/gejala metastasis

ht.
d. Semua
diatas benar

hu.

41. Koriokarsinoma bermetastasis terutama pada organ :

hv.

a. Vagina

hw.

b. Otak

hx.

c. Hati

hy.

d. Paru

jawaban

hz.42. Yang termasuk PTG dengan kriteria resiko tinggi, adalah :


ia.
ib.

a. Durasi < 4 bulan


b. Tidak
aterm

didahului

ic.
hamil

c.
Sudah
kemoterapi

id.

mendapat

d. hCG < 40.000

ie. 43. Pada PTG stadium II (FIGO)


if.

a. Menyebar ke paru

ih.

ig.

b. Menyebar ke tempat lain

c. Keluar uterus masih di


pelvis

ii.

d. Lesi terbatas di uterus

ij. 44. Prinsip penatalaksanaan PTG adalah :


ik.

a.
kinis

Diagnostik

il.

b. Tak perlu
sebelum terapi

PTG

secara

histopatologi

im.
c. Kedua jenis PTG
diterapi dengan cara yang sama
in.

d. Semua benar

io. 45.Kesehatan reproduksi adalah keadaan kesejahtraan fisik, mental dan


sosial yang utuh yang berhubungan dengan :
ip.

a. Proses kehamilan, persalinandan dan nifas di masa remaja

iq.

b. Proses reproduksi dan komplikasinya di masa remaja

ir.

c. Siklus reproduksi, fungsi serta proses2nya pada masa remaja

is.

d. Fungsi reproduksi dan segala permasalahannya pada masa


remaja

it. 46. Masa remaja terjadi pada usia :


iu. 47. Masalah yang menimpa remaja antara lain :
iv. 48. Masalah sexs yang sering dihadapi oleh masa remaja :
iw. 49. Resiko hubungan sexs pada usia muda :
ix. 50. Pembekalan informasi yang diperlukan oleh remaja :
iy.

a.
Perkembangan
fisik,
kegiatan
dan
pematangan
sexsual

iz.

b. Proses reproduksi
bertanggun jawab

yang

jc. 51. Untuk melihat anatomi tuba :

ja.

c. Pergaulan
antar remaja

yang

sehat

jb.

d. Semua jawaban di atas


benar

jd.

a. Laparoskopi

jf.

c. Endoskop

je.

b. Histerosalfingografi

jg.

d. Kolposkopi

jh. 52. Untuk melihat keadaan lumen tuba di perlukan pemeriksaan :


ji.

a. Laparoskopi

jk.

c. Endoskop

jj.

b. Histerosalfingografi

jl.

d. Kolposkopi

jm.

53. Bila kedua tuba non paten maka tindakan :

jn.

a. Intra uterin inseminasi

jp.

c. Senggama terjadwal

jo.

b. Picu opulasi

jq.

d. In vitro vertilisasi

jr. 54. Secara klinis sikap pertama menghadapi kasus amenore primer
memberi preparat :
js.

a. Estrogen

jv.

d. Squantial EP

jt.

b. Progesteron

jw.

e. Androgen

ju.

c. Kombinasi EP

jx. 55. Pada dasarnya penanganan perdarahan uterus disfungsional pada


wanita dengan selaput dara intak, maka menghentikan perdrhn dgn:
jy.

a. kuret

kb.

d. anti koagulasi

jz.

b. preparat estrogen

kc.

e. Kombinasi EP

ka.

c. preparat progestron

kd. 56. Salah satu cara untuk mengetahuiovulasi adalah Dating


Endometrium dimana spesimen endometrium diambil pada hari ke
(dihitung dari hari pertama menstruasi) :
ke.

a. 8 - 11

kh.

kf.

b. 12 - 15

ki.

kg.

d. 20 23
e. 24 27

c. 16 - 19

kj. 57. Pada pemeriksaan ginekologi untuk follow up akseptor. Alat


kontrasepsi dalam rahim pemeriksaan pertama yang dilakukan adalah :
kk.

a. Vaginal toucher

kn.

kl.

b. Inspekulo

ko.

km.

c. Palpasi abdomen

d. Ultrasonografi
e. Meraba tali AKDR

kp. 58. Seorang gadis mengeluh perdarahan pervaginam


menstruasi dan tidak teratur. Maka kondisi tersebut :
kq.

a. Polimenore

kr.

b. Hipomenore

ks.

c. Oligomenore

kt.

di

luar

d. Hipermenore

ku.

e. Metrorhagia

kv.59. Perdarahan yang terjadi saat menstrusi berhubungan dengan :


kw.
a. Tinggi dan lamanya
kadar estrogen
kx.

b. Tinggi dan lamanya kadar


progesteron

ky.

c. Withdrawl LH

kz.

d. Withdrawl progesteron

la.

e. Withdrawl FSH

lb. 60. Pilihan yang paling benar tentang hipotesa threshold windows:
lc.

a. Kadar FSH 10% - 30%


diatas kadar minimal dalam
kurun waktu tertentu

lf.

d. kadar FSH 10-30 % diatas


kadar minimal dan elemen
waktu

ld.

b. kadar FSH 10-30 % dioatas


kadar minimal

lg.

e. Tidak ada pengaruh FSH


dan Elemen waktu

le.

c. kadar FSH lebih rendah


dari kadar ambang minimal

lh. 61. Dengan kontrasepsi dimaksudkan, kecuali:


li.

a. Dapat menunda kehamilan


berikutnya

lk.

c.
Dapat
kelahiran

menjarangkan

lj.

b.
Dapat
kehamilan

ll.

d. Dapat
kesuburan

menghentikan

merencanakan

lm. 62. Pada masa perimenopouse setelah gangguan haid biasanya diikuti
dengan :
ln.

a. Osteoporosis

lp.

c. Gangguan vasomotor

lo.

b. Gangguan kardiovaskuler

lq.

d. Atrofi kulit

lr. 63. Pada menopouse keadaan FSH mencapai :


ls.

a. > 5 MLU/ml

lu.

c. > 20 MLU/ml

lt.

b. > 10 MLU/ml

lv.

d. > 30 MLU/ml

lw. 64. Menopouse prekok dimulai usia :

lx.

a. < 40 tahun

lz.

c. < 50 tahun

ly.

b. < 45 tahun

ma.

mb.

65. Keluhan jangka panjang timbul setelah :

d. < 55 tahun

mc.

a. Tidak haid > 3 bulan

me.

c. Tidak haid > 8 bulan

md.

b. Tidak haid > 6 bulan

mf.
bulan

d. Tidak haid > 12

mg.

66. Dalam 1 tahun tidak haid densitas tulang berkurang :

mh.

a. 2%

mj.

c. 4%

mi.

b. 3%

mk.

d. 5%

ml. 67. pada


sebanyak:

remodeling

tulang

akan

mengandung

bhn

mm.

a. 20-30 %

mo.

c. 60-70 %

mn.

b. 40-50 %

mp.

d. 80-90 %

mq.

organik

68. estrogen mempengaruhi :

mr.

a. ostoklas

mt.

c. jaringen kolagen

ms.

b. osteoblas

mu.

d. air

mv. 69. dalam skala resiko osteoporosis, bila tdk haid selama 12-24 bulan
mendpt nilai :
mw.

a. 1

my.

c. 3

mx.

b. 2

mz.

d. 4

na.

70. dalam skala resiko osteoporosis, pengguna alkohol mendpt nilai:

nb.

a. 1

nd.

c. 3

nc.

b. 2

ne.

d. 4

nf. 71. Pada menopouse tanpa uterus TSH :


ng.

a. kombinasi E dan P

ni.

c. P saja

nh.

b. E saja

nj.

d. Bergantian E dan P

nk.
Seorang wanita 20 tahun, belum nikah, mengeluh haidnya selalu
maju 10 hari, lama haid 5 hari, sehari ganti pembalut 3x :
nl. 72. Diagnose kasus :

nm.

a. Oligomenorea

no.

c. Hipermenorea

nn.

b. Polimenorea

np.

d. Hipomenorea

nq.
nr.

73. Kemungkinan yang terjadi :

a.
Stadium
memanjang

ns.

folikuler

b. Stadium luteal memanjang

nt.
c.
Stadium
proliferasi memendek
nu.

d. Siklus anovulatoir

nv.74. terapi yg diberikan :


nw.

a. p

ny.

c. Picu ovulasi

nx.

b. E - P

nz.

d. kuretase

oa.

75.

ob.

76. Terapi yang diberikan untuk di atas :

oc.

a. Progesteron

od.
b.
Progesteron
og. 77. Bila
dilakukan :
oh.
oi.

oe.

Estrogen
terapi

diatas

tidak

a. Kuretase

b. Pemberian
asetat

of.

d. Transfusi

menunjukkan
oj.

leoprolin

c. Kuretase

hasil

tindakan

yang

c. Histerektomi

ok.
d.
menopause

Konservatif

ol. 78. Suami istri menikah 3 tahun tapi belum punya anak. Umur istri 24
tahun haid teratur. Pemeriksaan fisik istri tidak ada kelainan, hasil sperma
analisis : Oligo. Pemeriksaan berikutnya :
om.

a. Hormonal

on.
b.
ulangan

Sperma

analisis

oo.

c. Laparoskopi

op.

d. Histerosalpingografi

oq.
79. Bila hasil pemeriksaan sperma analisa normal, tindakan yang
dilakukan :
or.

a. Induksi dengan klomifen


sitrat

os.
b.
senggama

Uji

pasca

ot.
ou.

c. Laparoskopi
d. Histerosalpingograf

ov.80. Jika hasil laparoskopi menunjukkan kelainan:


ow.

a. IUI

oy.

c. Repair tuba

ox.

b. IVF

oz.

d. Histeroskopi

pa.
pb.
pc.
pd.

SOAL MIDTERM OBSGIN 2

1. yang memegang peranan dalam ovulasi ;


a. hipotalamus
c. ovarium
b. hipofise
d. semua benar
e. 3. pada fase menstruasi terjadi peristiwa:
a. regenerasi epitel
d. disintergrasi dan
endotmetrium
autolisis sel epitel dan
b. kelenjar berkelok2
stroma endometrium
c. stroma tumbuh
e. 5.bila tdpt bercak perdarahan antara 2 haid kemungkinan:
a. menoragi
c. menarche
b. metroragi
d. amenore
e. 6. setelah ovulasi folikel pecah akan menjadi:
a. korpus albican
c. folikl d graaf
b. korpus luteum
d. ovarium kistik
e. 7. pola hipotalamus-hipofise-ovarium meningkat maka akan
menekan :

a. LH
c. PROLAKTIN
b. GH
d. FSH
e. 8. Pada siklus haid, pengaruh peningkatan hormone estrogen pada
endometrium
f.
g.

a.
Disintegrasi
endometrium
b. perkembangan folikel

epitel

h.
i.

c. otolisis epitel endometrium


d.
proliferasi
endometrium

j. 12. secara klinis sikap pertama untuk kasus amenore :


a.
b.
c.
f.

estrogen
d. sequential EP
progesterone
e. androgen
kombinasi E/P
14. perdarahan saat menstruasi erhubungan dengan :

kelenjar

a. estrogen tinggi dan


lama
b. progesteron tinggi
f. 16. Pseuomiksoma peritoneum sering

c. Withdrawl LH
d. Withdrawl progesteron
e. Withdrwal FSH
terjadi pada :

a.
b.
c.
f.

kista ovarium musin


d. kista folikel
kista ovarium serus
e. salah semua
kista lutein
17. dibawah ini termasuk kista ovarium non neoplastik, kec:

a.
b.
c.
f.

kista lutein
d. kita dermoid
kista folikel
e. smua benar
polikistik ovarium
21. mioma uteri terhadap kehamilan membawa dampak sbb,
kecuali :

a. tak berpengaruh apa2


b. dapat menyebabkan
abortus
c. dapat menyebabkan
kelainan letak
f. 23. tumor Brenner merupakan :

d. persalinan terhalang bila


mioma di fundus dan
menekan saluran tuba
e. semua benar

a. tumor kista ovarium non


c. tumor solid non neoplastik
neoplastik
d. tumor slid ovarium
b. tumor kista ovarium
neoplastik
neoplastik
e. Semua salah
f. 24. seorang wanita umur 35 tahun mengeluhkan ada benjolan,
gangguan haid dan 2 kali keguguran
g. 25. untuk memastikan kasus tersebut diperlukan diagnose
a.
b.
c.
f.

Pemeriksaan dalam
d. Semua benar
Usg
e. Semua salah
Kuretase
Mioma uteri merupakan tumor jinak yang biasa dari otot dan
jaringan ikat uterus, berdasarkan hasil biopsy oleh NOVAKS
dilaporkan bahwa :

a. 27 % pada wanita
menarch
b. 27 % pada wanita 25 th
c. 27 % timbul setelah
menopause
f.
g. Screening pap adalah

d. 27 % dari wanita
Indonesia
e. 27 % wanita kulit putih

a. Screening awal seksual


aktif
b. Pasca kistektomi tanpa
keganasan tiap 3 tahun

c. Pasca kistektomi NIS


tiap 3 bulan selama 2
tahun
d. A&c bener
e. A,b,c bener
f. 30. Ibu primigravida 16 tahun usia kehamilan 18 minggu fisiologi dng
tumor ovarium solid, penanganannya adalah
a. Tunggu kehamilan aterm
d. Periksa petanda tumor CEA
b. Laparotomi berencana
125
c. Tunggu
partus
kmdn
e. Punctio tumor ovarium
laparotomi
b. 33. Deteksi dini kasinoma ovarium belum dapat dilakukan dengan
memuaskan sehubungan dengan :
a.
b.
c.
d.

Lokasi anatomis dalam kavum pelvis


Besar tumor tidak menunjukkan tingkat keganasan
Belum ada tumor markaer
Sering pada usia reproduksi

c.
e.
f.
g.
3.
4.

5.
6.

7.

d.
SOAL OBGYN

2 Ibu usia 30 tahun tes post icoital normal:


a.
Picu ovulasi
c.
Laparoskopi *
b.
Clomifen sitrat
Pada infeksi vagina yang menyebabkan infertil:
a.
Efek spermatisida
c.
Plengketan
b.
Efek anti senggama *
d.
Penyempitan
Transmisi vertical HIV paling besar kemungkinan terjadi
saat:
a.
Antenatal
c.
Postpartum
b.
Intrapartum
d.
Laktasi *
Darah haid yang menumpuk sampai tuba disebut:
a.
Hematometra
c.
Hematosalping *
b.
Hematokolpos
d.
Hematuri
Penggunaan spermatisida berupa tablet vaginal bekerja:
a.
Langsung setelah
c.
1 jam setelah
dimasukkan
dimasukkan *
b.
1/2 jam setelah
d.
2 jam setelah
dimasukkan
dimasukkan
Tanda tanda stabilisasi baik adalah:
a.
Kesadaran membaik
c.
Produksi urin >100 cc
b.
Tekanan darah sistol
setiap 4 jam
meningkat *
d.
Semua benar

8.

Garis yang dibuat 0,05 0C sampai 10C dari suhu tertinggi

disebut:
a.
Garis aman
c.
Garis pelindung *
b.
Garis subur
d.
Garis batas
9.
.HIV adalah penyakit infeksi virus yang menyerang :
a.
Eritrosit
c.
CD4 *
b.
Leukosit
d.
Trombosit
10.
Resiko terjadinya transmisi perinatal sebanding dengan :
a.
Kadar CD4
d.
Terjadi infeksi
b.
Viral load *
opportunistic
c.
Stadium HIV
11.
Pada folikel, sel granulosa akan menghasilkan :
a.
Estrogen *
c.
Androgen
b.
Progesteron
d.
LH
12.
Interval haid kurang dari 21 hari :
a.
Oligomenore
c.
Hipermenore
b.
Polimenore *
d.
Hipomenore
13.
Pernah haid, kemudian tidak haid tapi belum memenuhi
amenore:
a.
Amenore primer
c.
Menstruasi prekoks
b.
Pseudomenore
d.
Delayed menstruasi *
14.
Pada amenore hiperprolaktinemia diberikan:
a.
Bromokriptin *
c.
Progesteron
b.
Etinil estradiol
d.
GnRH
15.
Pada penderita metrorhagia yang sudah menikah, pilihan
pertama pengobatan :
a.
Estrogen
c.
Kuretase *
b.
Progesteron
d.
Histerektomi
16.
Pada infeksi vagina, yang menyebabkan infertil :
a. Efek spermisida
c. Perlengketan
b. Efek anti senggama *
d. Penyempitan
17.
Syok yang diakibatkan oleh berkurangnya tonus vaskuler
akibat infeksi disebut
sebagai syok :
a. Septik *
c. Hipovolemik
b. Neurogenik
d. Kardiogenik
18.
Dalam praktik kebidanan jenis syok yang paling banyak
menyebabkan kematian ibu adalah syok :
a.
Septik
c.
Septik dan hipovolemik
b.
Hipovolemik
*
d.
Kardiogenik
19.
.Pencegahan transmisi perinatal dilakukan dengan :
a.
Menurunkan maternal
c.
Menurunkan paparan
viral load
janin terhadap secret ibu
b.
Menurunkan transfusi
d.
Semua jawaban diatas
maternal fetal
benar *
20.
Pada folikel dominan suasana dalam folikel bersifat :

a.
b.

Androgenik
c.
Progesteron
Estrogenik *
d.
LH dominan
21.
Pada ovulasi terjadi puncak :
a.
FSH
c.
Progesteron
b.
LH *
d.
Estrogen
22.
Perubahan pada endometrium yang periodenya selalu
tetap adalah :
a.
Fase proliferasi
c.
Fase sekresi *
b.
Fase folikuler
d.
Fase menstruasi
23.
Pada periode seleksi folikel dominan yang paling
berperan adalah :
a.
LH
c.
Progesteron
b.
Estrogen
d.
FSH *
24.
Amenore sekunder :
a.
Tidak haid 3 bulan
c.
Tidak haid 3 bulan / 6 bulan
b.
Tidak haid 3 siklus *
d.
Tidak haid 3 bulan / 9 bulan
e.
25.
Amenore primer :
a.
Usia 14 tahun, sex
c.
Usia 16 tahun, sex
sec (+)
sec (+) *
b.
Usia 16 tahun, sex sec
d.
Usia 18 tahun, sex sec
(-)
(+)
26.
Ibu hamil dengan HIV perlu diberikan ART. Rekomendasi
saat pemberian adalah
a. Sejak diketahui HIV (+)
c. Sejak usia kehamilan > 16
b. Sejak usia kehamilan > 13
minggu
minggu *
d. Sejak usia kehamilan > 20
minggu
27.
Selama menyusui (hiperprolaktinemia) pada ovarium :
a. Tidak terjadi pertumbuhan
c. Menghasilkan progesterone
folikel *
d. Berespon terhadap
b. Menghasilkan estrogen
gonadotropin
28.
Metode amenore laktasi tidak dapat digunakan lagi bila :
a. Ibu bekerja
d. Tidak menggunakan
b. Terpisah dengan bayi
pengganti ASI
c. Bayi berusia 6 bulan *
29.
Pada uji mucus serviks, bila terdapat cabang pertama
dan kedua nilainya :
a. 0
c. 2 *
b. 1
d. 3
30.
Pemeriksaan lab yang diperlukan pada pasien dengan
syok adalah :
31.
a. Golongan
33.
c. Ureum /
darah
kreatinin
32.
b. Hb
34.
d. Semua
benar *

35.
36.
37.
38.

SOAL AKBID
39.
1. Hasil perkawinan monohybrid yaitu persilangan dengan satu sifat
beda, bunyi hukum ini di kenal dengan hukum pemisahan gen yang
sealel adalah
a) Hukum Mendel I
d) Hukum Mendel II dan III
b) Hukum Mendel II
e) Hukum Mendel III
c) Hukum Mendel I dan II
f) Hukum Mendel IV
2. Dalam suatu persilangan antara ercis biji bulat dengan tanaman ercis
biji berkerut didapatkan hasil ercis biji bulat F1. Kemudian apabila F1
dengan F1 di lakukan persilangan, berapa ercis biji berkerut yang di
dapatkan.
a. 5
b. 3
c. 2

d. 1
e. 6

3. Kadang kadang sifat individu perkawinan tidak didominasi oleh salah


satu induknya. Dengan kata lain sifat dominan tidak muncul secara
penuh, peristiwa ini menunjukkan sifat
a. Intragentotif
b. Interfenotif
c. Intermediet

d. Intramediet
e. Ekstrafenotif

4. Macam dan jumlah gamet dapat ditentukan dengan menggunakan


rumus agar memudahkan dalam menghitung jumlah gamet dengan
konstanta n adalah jumlah gen heterozigot. Di bawah ini rumus
tersebut adalah
a) 2n
b) 3n
c) 4n

d) 5n
e) 1n

5. Persilangan dengan dua sifat beda yang dikenal dengan nama


persilangan dihibrid adalah bunyi dari hukum
a. Hukum Mendel I
b. Hukum Mendel II
c. Hukum Mendel III

d. Hukum Mendel IV
e. Hukum Mendel I dan II

6. Hormon yang bekerja secara langsung pada


merangsang produksi sperma, adalah :

spermatogonia untuk

a. Testosteron
b. FSH
c. LH

d. Hormon pembebas
e. Hormon sel leydig

7. Hormon yang bekerja secara tidak blangsung pada spermatogonia untuk


merangsang produksi sperma, adalah :
a. Testosteron
b. FSH
c. LH

d. Hormon pembebas
e. Hormon sel leydig

8. Berikut adalah tidak benar mengenai proses ereksi dan ejakulasi ;


a. Ereksi terjadi di bawah pengendalian saraf otonom
b. Ejakulasi merupakan mekanisme keluarnya cairan sperma
c. Rangsangan sympatis menyebabkan vasodilatasi arteri yang
menyebabkan penis ereksi
d. Setelah ejakulasi, spermatozoa dapat hidup 24-72 jam dalam organ
reproduksi wanita
e. Rangsang parasimpatis menyebabkan otot bulbokavernosusm
berkontraksi secara berirama, menyebabkan cairan semen keluar.
9. Bagian dari alat reproduksi laki-laki yang juga merupakan organ endokrin
karena menghasilkan hormone, adalah :
a. Scrotum
b. Testis
c. Vas deferens

d. Prostat
e. Glandula bulbourethralis

10. Hal-hal dibawah ini yang salah mengenai analisis semen manusia,
yaitu :
a. Syaratnya adalah seorang dewasa, sehat jasmani dan rohani
b. Secara makroskopis dapat dilihat kekentalan, bau, warna dan volum
per ejakulat
c. Secara mikroskopis dapat dilihat kecepatan, motilitas dan bentuk dari
sperma
d. Botol yang digunakan uintuk menampung sperma tidak boleh
berbahan plastic
e. Bau khas sperma seperti bunga akasia
11. Fisiologi reproduksi perempuan jauh lebih kompleks daripada laki;laki.
Dikatakan demikian karena hal-hal dibawah ini, kecuali :
a. Menghasilkan sel telur
b. Menerima sperma dari laki-laki
c. Memberikan keadaan yang cocok untuk terjadinya fertilisasi
d. Mampu memberi makanan bayi yang sedang berkembang, sebelum
maupun sesudah fertilisasi
e. Bertanggung jawab terhadap kelangsungan hidup sperma
12. Bagian yang berperan sebagai tempat terjadinya fertilisasi adalah :

a. Ovarium
b. Tuba uterine
c. Uterus

d. Vagina
e. Vestibulum vagina

13. Berikut adalah salah mengenai pembentukan sel telur dan siklusnya,
yaitu :
a. Pada pubertas terjadi pematangan folikel primordial menjadi folikel
primer di bawah pengaruh hormone gonadotropin
b. Pelepasan oosit matang dari folikel de Graaf disebut ovulasi
c. Folikel primer menghasilkan hormone estrogen sambil terus
berkembang menjadi folikel sekunder
d. Folikel sekunder berkembang menjadi corpus luteum yang
menghasilkan hormone estrogen dan progesterone
e. Regresi pada endometrium terjadi karena kadar estrogen dan
progesterone yang menurun tajam dalam darah
14. Hormon yang mempertahankan korpus luteum saat terjadinya
pemuahan atau fertilisasi adala :
a. Estrogen
b. Progesteron
c. Corionic gonadotropin

d. Androgen
e. FSH

15. Hormon yang berperan pada saat persalinan dan sesudahnya, yang
menstimuli kontraksi mioepitel kelenjar mammae sehingga susu mengalir
ke putting, yaitu :
a. Estrogen
b. Progesteron
c. Oksitosin

d. Corionic Gonadotropin
e. FSH dan LH

16. Proses tertanamnya balastosis pada dinding endometrium yang tebal


disebut :
a. Embrio
b. Fetus
c. Implantasi

d. Impansi
e. Ekspansi

17. Berikut ini yang tidak berhubungan dengan proses kehamilan, adalah :
a. Dimulai dengan terbentuknya zygote
b. Sel sperma akan mengeluarkan 4 macam enzim utama untuk
menembus lapisan sel telur
c. Setelah sperma satu masuk, sel telur akan membuat proteksi agar
sperma-sperma berikutnya tidak dapat menembus sel telur
d. Pada masa embrio, organisme sangat peka terhadap apa saja yang
menggagu perkembangannya
e. Pada masa fetus, organisme jauh kurang rentan terhadap aksi zat
teratogen daripada embrio

18.

Perubahan sistem sirkulasi pada bayi dimulai saat:

a. Saat kontraksi adekuat


b. Ketuban pecah
c. Kepala bayi keluar

d. Saat bayi menangis


e. Saat dipotongnya tali pusar

19. Hormon yang berperan menyebabkan uterus berkontraksi dengan kuat


sehingga masa pra-bersalin dimulai, adalah :
a. Estrogen
b. Oksitosin
c. Prostaglandin

d. Estrogen dan oksitosin


e. Oksitosin dan prostaglandin

20. Bagian tersempit dari tuba uterine, yang sering merupakan tempat
kejadian dari kehamilan ektopit (KET), yaitu :
a. Infundibulum
b. Ampula
c. Isthmus

d. Pars interstitial
e. Corpus

21. Bagian dari genetalia interna perempuan yang homolog dengan testis,
yaitu ;
22.

a. Uterus

23.

b. Ovarium

24.

c. Tuba uterine

25.
minor

d.

Labia

mayor

26.

e. Mons veneris

dan

27. Pernyataan di bawah ini yang sesuai dengan definisi sel manusia
adalah
a. Unit terkecil dan penyusun
makhluk hidup
b. Unit dan penyusun makhluk
hidup
c. Memiliki struktur seperti
hewan

d. Terdiri atas banyak tipe dan


bermitosis
e. Memiliki susunan genetik
multipel

28. Bagian dari sel yang memiliki fungsi sebagai pengatur genetik karena
mengandung DNA adalah
a. Retikulum endoplasma
b. Nukleus
c. Membrane sel

d. Mitokrondia
e. Apparatus golgi

29. Organel yang memiliki 2 tipe struktural yang merupakan system


delivery internal cel yaitu

a. Retikulum endoplasma
b. Nukleus
c. Membrane sel

d. Mitokrondia
e. Apparatus golgi

30. Mitorondia sebagai organel sel sangat penting peranannya, pernyataan


di bawah ini yang mendukung hal tersebut adalah
a. Produksi energy
b. Tidak memiliki membrane
c. DNA tidak di temukan

d. Membrane tunggal
e. Fungsi respirasi sel tidak
termasuk di dalamnya

31. Fungsi dan peranan apparatus golgi di dalam sel penting kaitannya
dalam hal,
a. Menyimpan protein
b. Modifikasi karbohidrat
c. Transportasi molekul lemak

d. Keseimbangan asam basa


e. Transportasi elektrolit

32. Dasar genetika sel mencakup beberapa istilah di dalamnya, yang


berperan dalam pengatur karakter fisik dan psikis adalah
a. Kromosom
b. Gen
c. RNA

d. DNA
e. Sintesa Protein

33. Gen terdiri dari beberapa rantai yang mengatur protein dan rangkaian
genetik, yang berperan menerjemahkan nukleotida menjadi protein yaitu,
a. RNA
b. DNA
c. Kromosom

d. Intron
e. Ekson

34. Rangkaian genetika yang berfungsi mengontrol sifat biologis dan


genetis sel serta memiliki DNA yang berkaitan dengan aktivitas molecular
adalah
a. RNA
b. DNA
c. Kromosom

d. Intron
e. Ekson

35. Pengelompokan kromosom berdasarkan letak sentromer dibedakan


menjadi, kecuali
a. Telosentrik
b. Akrosentrik
c. Submetasentrik

d. Telometrik
e. Metasentrik

36. Pusat kromosom yang merupakan daerah terang dengan nama lain
kromosom kepala dan tidak mengandung gen merupakan definisi dari

a. Kromosom
b. Gen
c. Alel

d. Sentromer
e. Genosom

37. Kromosom yang berperan dalam menentukan jenis kelamin (genosom)


dalam tiap individu terdapat 2 jenis genosom yaitu
a. Y dan V
b. X dan Y
c. X dan Z

d. Y dan Q
e. X dan K

38. Kromosom tubuh manusia (autosom) berpasangan dan sangat


berperan dalam menentukan patologis atau tidaknya tubuh manusia,
jumlah yang benar yaitu
a. 22
44
b. 22
46
c. 22
40
39.

pasang kromosom atau


buah
pasang kromosom atau
buah
pasang kromosom atau
buah

d. 23 pasang kromosom atau


48 buah
e. 23 pasang kromosom atau x
dan y

Pernyataan tentang DNA di bawah ini yang benar, kecuali

a. Terdiri atas 2 rantai


polinukleotida
b. Tersusun secara double helix
c. Pada sel prokariotik
mitokondria dan plastida
unsirkuler

d. Pada sel eukariotik benang


tidak bercabang
e. Antara basa nitrogen
dihubungkan oleh ikatan
hidrogen

40. DNA merupakan makromolekul kompleks yg terdiri atas tiga macam


molekul, diantaranya di bawah ini, kecuali
a. Asam folat
b. Basa nitrogen
c. Pirimidin dan purin

d. Gula deoksiribosa
e. Protein deoksiribosa

41. DNA memiliki beberapa fungsi dalam pengaturan genetic, fungsi


tersebut di bawah ini yang benar, kecuali
a. Membentuk DNA
b. Sintesa protein
c. Control aktivitas sel

d. Pembawa informasi genetic


e. Sintesa RNA

42. Hasil sintesa atau transkripsi dari DNA dan mengandung D-ribosa, basa
nitrogen dan posfat adalah
a. DNA

b. RNA

c. Gen
d. Kromosom

e. Protein

43. Dalam fungsinya RNA di bagi menjadi beberapa tipe, tipe RNA di
bawah ini yang benar , kecuali
a. rRNA
b. mRNA
c. tRNA

d. eRNA
e. mRNA dan rRNA

44. Tipe RNA yang berfungsi sebagai pengatur sintesa protein dan
penyusun ribosom serta jumlahnya 80% dari jumlah RNA dalam sel
adalah
a. rRNA
b. mRNA
c. tRNA

d. eRNA
e. mRNA dan rRNA

45. Tipe RNA yang dibentuk dalam nucleus ydan tergantung pada kode
genetic DNA yang dibawa dengan ukuran yang bervariasi adalah
a. rRNA
b. mRNA
c. tRNA

d. eRNA
e. mRNA dan rRNA

f.
g.
h.
i.
j.
k.
l. SOAL LANJUTAN OBSGYN 2007
m.
n. 38. Salah satu penyebab menopause prekok:
o.
a. unilateral,
q.
c. tubektomi bilateral
salpingoporekromi
r.
d. total abdominal
p.
b. total abdominal
histerektomi
histerektomi
bisalpingoverektomi
s. 41. Faktor yang tidak berpengaruh terjadinya kehamilan kembar
adalah
t.
a. factor paritas
v.
c. factor umur
u.
b. factor riwayat
w.
d. factor keturunan
infertilitas
x.
e. factor bangsa
y. 42. Pernyataan mengenai kehamilan kembar dizigot adalah
z.
a. berasal dari 1 telur
ab. c. terdapat pada 1/3
aa. b. rupa dan sifat sama
kasus gamelli

ac. d. mempunyai 2
plasenta, 2 korion, dan 2
amnion

ad.

e. rupa dan sifat sama

ae.
45. Bertambahnya darah dalam kehamilan mencapai puncak
waktu umur kehamilan
af.

a. 40-42 minggu

ai.

d. 28-30 minggu

ag.

b. 38-40 minggu

aj.

e. 26-28 minggu

ah.

c. 32-36 minggu

ak.

66. Cara mendiagnosis DM dalam kehamilan

al.
urin

a. cukup priksa glukosa

am.

b. pemeriksaan GDP

an.
c. pemeriksaan Glukosa
darah post prandial
ao.

ap.
Dm Bumil Bila tidak diterapi :

69. Komplikasi

aq.
a. Sembuh sendiri bila
sudah partus

as.
c. janin dapat IUFD

ar.

at.
d. si ibu menjadi malnutrisi

b. janin KMK

au.
av.
aw.
ax.
ay.

SOAL OBGIN
az.

ba.

1.yang termasuk tanda pasti kehamilan

bb.

a.amenore

bc.

b.mengidam

bd.

c.tes kehamilan positif

bg.

2.termasuk diferensial diagnosis pada kehamilan

be.
d.dapat diraba dan
dikenali bagian janin
bf. e.ditemukan tanda Hegar

bh.

a.pseudosiesis

d.retensi urine

bi. b.kista ovarii


bj.c.mioma uteri

e.semua benar

bk.
3.tanda tidak pasti kehamilan yang ditandai dengan perubahan
warna kebiruan pada vagina dan servik
bl. a.Hegar

d.Brax nick

bm.

b.Chadwick

bn.

c.Piscaeseck

bo.

4.perubahan pada tubuh wanita hamil karena hormone

bp.

a.HCG

bq.

b.estrogen

e.Bundle

d.somatotrophin

br. c.progesteron
e.estrogen,progesterone,somatotropin
bs.5.yang tidak termasuk pedoman pemerikasaan obstetric
bt.
- seorang dokter berusaha memberikan pelayanan
kepada pasien dan keluarga
bu.
- menempatkan kepentingan pasien lebih utama dari
kepentingan dirinya
bv.
- pemeriksaan diruangan tertutup,tanpa adanya orang
yang tidak berkepentingan
bw.

- dokter harus ditemani perawat saat memeriksa pasien

bx.

- pemeriksaan dilakukan setelah perkenalkan diri

by.6.fertilisasai biasa terjadi di


bz.a
ca.b.pars ampularis tuba
cb.

c
d.infundibulum

7.Setelah fertilisasi sperma mengalami

cc.a.kapasitasi

c.reaksi akrosomal

cd.

b.lisis

d.A dan C benar

ce.

8.setelah konsepsi 36 jam terbentuk(morula)

cf. 9.hormon B HCG dibentuk oleh


cg.

a.sitotrofoblas

c.epiblas

ch.

b.sinsitiotrofoblas

ci. 10.organ mayor dan sistem organ terbentuk minggu ke


cj. a.I
ck.

c.II
b.III

d.IV

cl. 11.implantasi normal plasenta terjadi


cm.

a.korpus anterior

cn.

b.fundus

c.korpus posterior
d.semua benar

co.12.pada wanita hamil karena hormon tiroid BMR meningkat rata-rata


cp.

a.5-10 %

d.20-25%

cq.

b.10-15%

e.20-30%

cr. c.15-20%
cs. 13.kadar FE untuk wanita hamil (%)
ct. a.10-30
cu.

d.20-25

b.10-15

e.20-30

cv. c.15-20
cw.

14.aliran darah plasenta menuju janin

cx.

a.arteri umbilicus

c.vena umbilikalis

cy. b.duktus venosus

e.A,C bnr

d.duktus arteriosus

cz. 15.fungsi ketuban kecuali


da.

a.melindungi dari trauma

d. agar bebas bergerak

db.
b.sumber makanan
maturitas janin
dc.

c.menjaga suhu

dd.

16.janin menelan pada bulan

de.

a.1

b.2

c.3

e.tidak da hub.dngn

d.4

e.5

df.
dg.

17.darah ibu

dh.
a. bercampur dengan
darah janin
di. b.masuk vena umbilikalis

dj.c.menyeberangi plasenta
secara acak
dk.
umbilical

d.kembali melalui

dl. 18.tujuan ANC


dm.
a.menyiapkan fisik dan
mental ibu
dn.
b.menyiapkan ibu ,dan
anak dalam
kehamilan,persalinan,nifas

do.
c.menyiapka ibu sehat
pada postpartum
dp.

d.semua benar

dq.

e.A,B benar

dr. 19.foramen ovale menghubungkan


ds.a.vena cava inferior dan vena
porta
dt.
kiri

b.atrium kanan dan

du.
c.ventrikel kanan dan
ventrikel kiri
dv.d.aorta dan arteri pulmonal

dw. 20.yang paling tinggi kadar oksigen pada janin adalah darah yang
mengalir pada
dx.

a.arteri hipogastrika

c.arteri coroner

dy.

b.arteri pulmonalis

d.umbilikalis

dz.21.usus janin mulai mengalami peristaltis pada umur


ea.

a.9 minggu

c.11

eb.

b.10

d.12

ec.22.pengaturan dan komposisi air ketuban


ed.

a.ginjal janin

ee.

b.menelan

c.selaput ketuban
d.benar

ef. 23.volume ketuban pada hamil fisiologis akan mengalami puncaknya


pada
eg.

a.34 minggu

eh.

b.36

d.40
e.42

ei. c.38
ej. 24.tes kocok pada air ketuban melalui amnionitis untuk menilai
ek.

a.matang ginjal janin

el. b.matang pencernaan janin

c.matang paru
d.matang darah

em.

25.satuan aktivitas uterus

en.

26.kekhasan otot polos uterus(lapisan otot beranyam)

eo.
27.lapisan otot uterus(lapisan luar longitudinal,tengah
lapisan otot beranyam,lapisan dalam sirkuler)
ep.

28.obstipasi disebabkan

eq.

29.sifat kontraksi uterus

er. 30.pelurusan badan janin tujuannya untuk


es.31.etiologi putar paksi dalam
et. 36.perasat
eu.

-distosia bahu yang bukan termasuk distosia

ev.-yang tidak dapat lahir pervaginam


ew.

-fase hipertonik

ex.

-kehamilan trimester II

ey. -ekspulsi terjadi pada


ez.38.yang dimaksud hamil trimester II adalah
fa. a.14-28 minggu
fb.

fc. c.16-28

b.12-28 minggu

fd. 39.bunyi jantung janin mulai terdengar dengan stetoskop usia


fe.40.frekuensi alat USG (3-5 MHz)
ff. 41.atenuasi(pengurangan intensitas gelombang suara)
fg. 42.sarat USG janin
fh. 43.indikasi USG
fi. -usia kehamilan tidak
jelas

fj. -kehamilan multiple

fk.-perdarahan dlm
kehamilan

fo.

Janin besar

fl. -kematian janin

fp.
polihidramnion,oligo

fm.

-kehamilan ektopik

fq.

fn.

-molahidatidosa

-dll

fr. 44.USG pada trimester 2-3 adalah untuk (letak dan


presentasi janin)
fs.45.USG pada trimester I untuk(usia kehamilan,penentuan
lokasi,detekdi embrio dan janin hidup,evalusi komplikasi
kamilan,deteksi kelainan bawaan,deteksi kehamilan
ganda,evalusi tumor pelvic,AKDR)
ft. 46.indikasi pemeriksaan USG(sama ma jawaban nmr 43)
fu. 47.keamanan alat USG
fv. -gelombang ultrasonic yang digunakan dalam bentuk pulsa so
efek kumulatifnya dalam jaringan sangat kecil
fw.
-dinding abdomen ibu akan mengabsorbsi sebagian
intensitas gelombang ultrasonic
fx.-pemakaian USG jenis real time,dapat menghindari
terfokusnya intensitas gelombang ultrasonic pada organ terlalu
lama
fy. -vaskularisasi dinding abdomen ibu dan tubuh janin akan
menetralisir efek panas dari gelombang ultrasonik
fz. 48.USG pada intensitas > 100 mw/cm dapat merusak
ga. 49.mekanisme pengurangan intensitas gelombang suara
disebut(atenuasi)
gb.

50.yang dimaksud konjugata vera

gc.
a.jarak tepi bawah
simfisis kepromontorium

gf.d.jarak anatara tepi atas


simfisis kepromontorium

gd.
cm

gg.
1,5 cm

b.konjugata vera + 1,5

ge.
c.jarak antara ke 2
spina ischiadika

e.konjugata obstetrika

gh. 51.perubahan warna kulit ibu hamil dengan hiperpigmentasi seperti


kloasma gravidarum dipengaruhi oleh hormone (MSH>Melanophore stimulating hormone)
gi. 52.hiperemis gravidarum pxb idiopatik factor yang mempengurahi
kecuali
gj. a.faktor organic
gk.

d.kehamilan ganda

b.f.psikologik

e.grande multi gravida

gl. c.primigravida
gm.
53.partograf adalah alat Bantu untuk monitoring
persalinan.partograf WHO berguna
gn.

a.adalah memantau kala I fase laten n aktif persalinan

go.

b.memantau kala I,II-IV persalinan

gp.
c.memantau kemajuan persalinan n membantu provider dalam
mengambil keputusan n penatalaksanaan,melakukan pencatatan saat
pembukaan servik lebih dari 4 cm fase aktif kala I
gq.

d.semua benar

gr. e.salah semua


gs.54.partograf untuk mengurangi
gt. a.kejadian seksio sesarea

gv.c.morbiditas n mortalitas
maternal n neonatal

gu.

b.partus lama

gw.

55.partograf hanya digunakan saat persalinan di

gx.

a.RSUP

gy.b.rumah bersalin p

d.benar semua
e.salah semua

gz.c.polindes/PKMS
ha.
60.Puerperium agar sembuh sama sebelum partus berapa
minggu
hb.

a.3

d.6

hc.

b.4

e.7

hd.

c.5

he.

61.pembukaan 8 dipantau berapa jam

hf.62.hisnya 2x dalam 10 menit 20 detik(pemberian


oksitosin drip)

hi.

hg.
SOAL OBGYN
hh.
1. Darah yang terkumpul di tuba di sebut:
a.
Hematometra
c.
Hematosalping
b.
hematokolpos
d.
Hematuri
e. 2. Ibu usia 30 tahun tes post icoital normal:
d.
e.

3.
b.
c.
4.
a.
b.
5.
saat:
e.
f.

Picu ovulasi
f.
Laparoskopi *
Clomifen sitrat
Transmisi perinatal sebanding dengan peningkatan:
CDU
d.
Infeksi opportunistic
Viral load
Pada infeksi vagina yang menyebabkan infertil:
Efek spermatisida
c.
Plengketan
Efek anti senggama *
d.
Penyempitan
Transmisi vertical HIV paling besar kemungkinan terjadi

Antenatal
g.
Postpartum
Intrapartum
h.
Laktasi *
6.
Darah haid yang menumpuk sampai tuba disebut:
e.
Hematometra
g.
Hematosalping *
f.
Hematokolpos
h.
Hematuri
7.
Penggunaan spermatisida berupa tablet vaginal bekerja:
e.
Langsung setelah
g.
1 jam setelah
dimasukkan
dimasukkan *
f.
1/2 jam setelah
h.
2 jam setelah
dimasukkan
dimasukkan
8.
Tanda tanda stabilisasi baik adalah:
e.
Kesadaran membaik
g.
Produksi urin >100 cc
f.
Tekanan darah sistol
setiap 4 jam
meningkat *
h.
Semua benar
9.
Garis yang dibuat 0,05 0C sampai 10C dari suhu tertinggi
disebut:
e.
Garis aman
g.
Garis pelindung *
f.
Garis subur
h.
Garis batas
10.
.HIV adalah penyakit infeksi virus yang menyerang :
e.
Eritrosit
g.
CD4 *
f.
Leukosit
h.
Trombosit
11.
Resiko terjadinya transmisi perinatal sebanding dengan :
e.
Kadar CD4
h.
Terjadi infeksi
f.
Viral load *
opportunistic
g.
Stadium HIV
12.
Pada folikel, sel granulosa akan menghasilkan :

e.
Estrogen *
g.
Androgen
f.
Progesteron
h.
LH
13.
Interval haid kurang dari 21 hari :
e.
Oligomenore
g.
Hipermenore
f.
Polimenore *
h.
Hipomenore
14.
Pernah haid, kemudian tidak haid tapi belum memenuhi
amenore:
e.
Amenore primer
g.
Menstruasi prekoks
f.
Pseudomenore
h.
Delayed menstruasi *
15.
Pada amenore hiperprolaktinemia diberikan :
e.
Bromokriptin *
g.
Progesteron
f.
Etinil estradiol
h.
GnRH
16.
Pada penderita metrorhagia yang sudah menikah, pilihan
pertama pengobatan :
e.
Estrogen
g.
Kuretase *
f.
Progesteron
h.
Histerektomi
17.
Pada infeksi vagina, yang menyebabkan infertil :
e. Efek spermisida
g. Perlengketan
f. Efek anti senggama *
h. Penyempitan
18.
Syok yang diakibatkan oleh berkurangnya tonus vaskuler
akibat infeksi disebut
sebagai syok :
e. Septik *
g. Hipovolemik
f. Neurogenik
h. Kardiogenik
19.
Dalam praktik kebidanan jenis syok yang paling banyak
menyebabkan kematian ibu adalah syok :
e.
Septik
g.
Septik dan hipovolemik
f.
Hipovolemik
*
h.
Kardiogenik
20.
.Pencegahan transmisi perinatal dilakukan dengan :
e.
Menurunkan maternal
g.
Menurunkan paparan
viral load
janin terhadap secret ibu
f.
Menurunkan transfusi
h.
Semua jawaban diatas
maternal fetal
benar *
21.
Pada folikel dominan suasana dalam folikel bersifat :
e.
Androgenik
g.
Progesteron
f.
Estrogenik *
h.
LH dominan
i.
22.
Pada ovulasi terjadi puncak :
e.
FSH
g.
Progesteron
f.
LH *
h.
Estrogen
23.
Perubahan pada endometrium yang periodenya selalu
tetap adalah :
e.
Fase proliferasi
g.
Fase sekresi *
f.
Fase folikuler
h.
Fase menstruasi
24.
Pada periode seleksi folikel dominan yang paling
berperan adalah :

e.
f.

LH
g.
Progesteron
Estrogen
h.
FSH *
25.
Amenore sekunder :
f.
Tidak haid 3 bulan
h.
Tidak haid 3 bulan / 6 bulan
g.
Tidak haid 3 siklus *
i.
Tidak haid 3 bulan / 9 bulan
26.
Amenore primer :
e.
Usia 14 tahun, sex
g.
Usia 16 tahun, sex
sec (+)
sec (+) *
f.
Usia 16 tahun, sex sec
h.
Usia 18 tahun, sex sec
(-)
(+)
27.
Ibu hamil dengan HIV perlu diberikan ART. Rekomendasi
saat pemberian adalah
e. Sejak diketahui HIV (+)
g. Sejak usia kehamilan > 16
f. Sejak usia kehamilan > 13
minggu
minggu *
h. Sejak usia kehamilan > 20
minggu
28.
Selama menyusui (hiperprolaktinemia) pada ovarium :
e. Tidak terjadi pertumbuhan
g. Menghasilkan progesterone
folikel *
h. Berespon terhadap
f. Menghasilkan estrogen
gonadotropin
29.
Metode amenore laktasi tidak dapat digunakan lagi bila :
e. Ibu bekerja
h. Tidak menggunakan
f. Terpisah dengan bayi
pengganti ASI
g. Bayi berusia 6 bulan *
30.
Pada uji mucus serviks, bila terdapat cabang pertama
dan kedua nilainya :
e. 0
g. 2 *
f. 1
h. 3
31.
Pemeriksaan lab yang diperlukan pada pasien dengan
syok adalah :
32.
a. Golongan
34.
c. Ureum /
darah
kreatinin
33.
b. Hb
35.
d.
Semua benar *
36.
37.
38.
39.

SOAL UJIAN TENGAH SEMESTER


OBSGYN II, Bln 10-11 2007

40.
1. Yang memegang peranan penting pada proses
ovulasi adalah:
41.
a.
Hipotalamus

42.
Hipofise

b.

43.
Ovarium

c.

44.
d.
Semua benar

45.
2. Berkaitan dengan siklus haid, hipotalamus
menghasilkan hormone:
46.
a.
Lutennizing
Hormone

48.
c.
Folikel
stimulating

47.
b.
Gonadotropin
Releasing
Hormone

49.
Estrogen

50.

3. Pada fase menstruasi terjadi peristiwa :

51.
a.
Regenerasi epitel
endometrium
52.
b.
Kelenjar
berkelok-kelok

55.
:

d.

53.
c.
Stroma tubuh
54.
d.
Disintegrasi
dan otolisis
sel-sel epitel
dan stroma
endometrium

4. Untuk menilai ovulasi dapat dipakai penilaian yaitu

56.
a.
Spinbarkheit

58.
c. Tes
hormonal basal

57.
d. Tes
suhu basal
badan

59.
d.
Semua benar

60.
5. Bila terdapat bercak-bercak perdarahan antara 2
haid kemungkinan termasuk dalam:
61.
a.
Menoragia

63.
c.
Menarche

62.
b.
Metroragia

64.
Amenore

d.

65.
6. Setelah proses ovulasi, folikel yang pecah akan
mungkin menjadi bentuk:

66.
a.
Korpus albican

68.
c.
Folikel de graaf

67.
b.
Korpus luteum

69.
Ovarium
polikistik

d.

70.
7. Pada proses hipotalamus hifopice ovarium kadar
estrogen yang tinggi dalam darah akan menekan
pengeluaran:
71.

a. LH

72.
b.
Growth hormon

73.
Prolaktin

c.

74.
FSH

d.

75.
8. Pada siklus haid pengaruh peningkatan hormon
estrogen pada endometrium:
76.
a.
Disintegrasi
epitel
endometrium
77.
b.
Perkembangan
folikel
80.

78.
c.
Otolisis epitel
endometrium
79.
d.
Proliferasi
kelenjar
endometrium

9. Yang termasuk sindrom pra haid:

81.
a.
Mudah
tersinggung
82.
Depresi

b.

83.
Migren

c.

84.
d.
Semua benar

85.
10. Gangguan haid yang termasuk dalam jumlah
darah yang keluar :
86.
a.
Hipermenore

88.
Spotting

c.

87.
b.
Hipomenore

89.
d.
Semua benar

90.
11. Haid terjadi oleh pengelupasan periodik endometrium
karena perubahan fisiologis hormonal. Peristiwa haid ini dipengaruhi oleh
organ:

91.
a.
Kortek serebri

93.
Hipofise

c.

92.
b.
Hipotalamus

94.
d.
Semua benar

95.
12. Secara klinis sikap pertama dalam menghadapi
kasus amenore primer adalah memberikan preparat:
96.
a.
Estrogen (E)

99.
d.
Squential EP

97.
b.
Progesteron (P)

100.
e.
Androgen

98.
c.
Kombinasi EP
101.
13. Pada dasarnya penanganan Perdarahan Uterus
Disfungsional (PUD) pada wanita dengan selaput darah masih intact
adalah menghentikan perdarahan dengan cara :
102.
Kuretase

a.

105.
Anti koagulasia

d.

103.
Preparat E

b.

106.
Kombinasi E/P

e.

104.
Preparat P

c.

107.
dengan :

14. Perdarahan yang terjadi saat menstruasi berhubungan

108.
a.
Tinggi dan lamanya kadar E

111.
Withdrawl Progesteron

d.

109.
b.
Tinggi dan lamanya kadar P

112.
Withdrawl FSH

e.

110.
Withdrawl LH

c.

113.
15. Seorang gadis mengeluh perdarahan pervaginam
diluar menstruasi dan tidak beraturan, maka kondisi tersebut:
114.
Polimenore

a.

115.
Hipomenore

b.

116.
Oligomenore

c.

117.
Hipermenore

d.

119.

16.

120.

17.

121.

18.

122.

19.

123.

20.

118.
Metrorhagia

e.

124.
21. Pengaruh mioma uteri terhadap kehamialan adalah
sebagai berikut, kecuali:
125.
a.
Mungkin tidak berpengaruh
apa-apa
126.
b.
Dapat menyebabkan abortus
127.
c.
Dapat menyebabkan kelaianan
letak

128.
d.
Persalinan dapat terhalang bila
mioma di fundus dan menekan
saluran tuba
129.
semua benar

e.

130.
22. Kista Stein-levental merupakan kista ovarium pada
wanita dengan gejala-gejala infertilita amenore dan obesitas. Ciri-ciri kista
ovarium:
131.
a.
Polikistik dengan permukaan
licin
132.
kapsul ovarium menebal

136.

b.

133.
Ovarium pucat

c.

134.
d.
Ovarium membesar 9-3 kali
135.
Semua benar

e.

23. Tumor Brenner merupakan :

137.
a.
Tumor kistik ovarium non
neoplastik
138.
b.
Tumor kistik ovarium neoplastik

139.
Tumor solid ovarium non
neoplastik

c.

140.
Tumor solid ovarium
neoplastik

d.

141.
Semua salah

e.

142.
24. Wanita 35 th, mengeluh ada benjolan, gangguan haid
dan 2 kali mengalami keguguran. Diagnose yang paling mungkin adalah :
143.
Mioma uteri

a.

146.
Tuba ovarial abses

d.

144.
Kistoma ovarii

b.

147.
Semua salah

e.

145.
Tumor brenner

c.

148.
25. Untuk memastikan diagnose kasus di atas perlu
dilakukan pemeriksaan anjutan seperti :
149.
Pemeriksaan dalam

a.

152.
Semua benar

d.

150.
USG

b.

153.
Semua salah

e.

151.
c.
Diatasi dan kuretase dengan
pemeriksaan perdarahan
154.
adalah:

26. Kista di daerah vagina sebagai akibat suatu radang,

155.
Kista gatner

a.

158.
Kista skene

d.

156.
Kista bartolin

b.

159.
Semua benar

e.

157.
Kista parauretra

c.

160.
27. Mioma uteri merupakan tumor jinak yang berasal dari
otot jaringan ikat uterus. Berdasarkan hasil otopsi oleh Novaks dilaporkan
bahwa:
161.
162.
a. 27%
ditemukan pada wanita menars

163.
b.
27% ditemukan pada wanita
25 th

164.
c.
27% masih tumbuh setelah
menopause

165.
d.
27% dari wanita Indonesia
166.
e.
27% dari wanita kulit putih

167.
28. Seorang mioma di uterus dengan berdiameter dari
korpus uteri. Dari servik:
168.
/ 0,9%

a. 0,5

170.
13 %

c. 10-

169.
3%

b. 1-

171.
20 %

d. 15-

172.
29. Pengaruh mioma uteri terhadap kehamilan yang mana
pada pasca perca persalinan dapat menyebabkan perdarahan pasca
persalinan primer, oleh mioma dapat menyebabkn:
173.
a.
Sulit melahirkan plasenta

176.
Sisa plasenta

d.

174.
Plansenta previa

177.
Infeksi post partum

e.

b.

175.
c.
Gangguan kontraksi rahim
178.
30. Ibu primigravida 16th usia kehamilan 18 minggu
fisiologi dengan tumor ovarium solid, penanganannya:
179.
Tunggu kehamilan aterm

a.

182.
d.
Periksa petanda tumor CEA 125

180.
Laparotomi berencana

b.

183.
Punctie tumor ovarium

181.
Tunggu partus kemudian
Laparotomi

c.

184.

e.

31. Skreening pap adalah:

185.
a.
Skreening pap awal saat seksual
aktif
186.
b.
Pasca histerektomi tanpa
keganasan setiap 3 tahun

187.
c.
Pasca histerektomi NIS atau
kanker invasive setiap 3 bulan
selama 2 tahun
188.
& B benar

d. A

189.
& C benar

e. A,B

190.
32. Untuk melakukan biopsi terarah pada lesi serviks
uterus yang dicurigai keganasan dipakai :
191.
Cairan Lugol 3%

a.

193.
Kolposkopi

c.

192.
Papaniculo test

b.

194.
Acetic Acid 3%

d.

195.
33. Deteksi dini kasinoma ovarium belum dapat dilakukan
dengan memuaskan sehubungan dengan:
196.
a.
Lokasi anatomis dalam kavum
pelvis

198.
c.
Belum ada tumor marker
patognomosis

197.
b.
Besar tumor tidak menunjukkan
tingkat keganasan

199.
Sering pada reproduksi

200.

34.

201.

35.

d.

202.
36. Disebut sebagai mola hidatidosa resiko tinggni
apabila ditemukan :
203.
a.
Kadar hCG > 50.000 IU/L

206.
d.
Histerektomi & MTX profilaksis

204.
b.
Pemberian MTX atau ACD

207.
e.
Evakuasi dan MTX profilaksis

205.
c.
Evakuasi dan pengawasan lanjut
208.
37. Sebagian besar penyakit trofoblas ganas pasca
mola hidatidosa, adalah:
209.
Mola parsial

a.

211.
Koriokarsinoma

c.

210.
Mola destruens

b.

212.
Embrional karsinoma

d.

213.

38. Dasar diagnosis PTG, adalah:

214.
a.
Titer hCG menetap/meningkat

216.
Tanda gejala metastasis

215.
b.
Perdarahan pervaginam yang
berulang (pasca evakuasi)

217.
d.
Semua jawaban diatas benar

218.
adalah:

c.

39. Yang merupakan ciri histopatologi mola hidatidosa,

219.
Stroma vili khorialis
hipervaskular

a.

220.
Stroma villi khorialis
avaskular

b.

221.
c.
Stroma villi invasi ke miometri
222.
Stroma villi mengecil

d.

223.
40. Salah satu gejala klinis yang khas pada mola
hidatidosa, adalah:
224.
Hiperemesis gravidarum

a.

225.
b.
Pembesaran uterus lebih
besar dari pada yang
seharusnya

226.
Perdarahan pervaginam

c.

227.
hCG (+)

d.

228.
229.
230.
231.
232.
233.
234.
235.
236.
237.
238.

SOAL UAS OBGIN SEMESTER 6 2007

1. Abortus
a. Penghentian kehamilan < 20 minggu
2. Abortus dini
a. < 12 minggu

239.
240.
3. Peristiwa abortus ditandai dilatasi servik tapi hasil
konsepsi masih ada
241.
a. abortus iminens : janin masih dalam rahim
tanpa dilatasi servik
242.

b. abortus inkomplet : pengeluaran sebagian

243.
c. abortus insipien : janin masih dalam
rahim dengan dilatasi servik
244.
semua

d. abortus komplitus : sudah dikeluarkan

245.
e. mised abortion : KJDR< 20 minggu, tidak
keluar selama 8 minggu atau lebih
246.

4. Indikasi abortus terapi etik

247.
gangguan
kesehatan ibu
atau
mengnacam
keselamatan ibu
248.
kehamilan akibat
251.

perkosaan atau
incest
249.
cacat janin
250.
retardasi mental

5. kehamilan ektopik

252.
Adalah kehamilan yang hasil konsepsinya tidak
berimplantasi dalam kavum uteri
253.

6. Trias yang khas pada kehamilan ektopik

254.
amenore

255.
nyeri perut
bawah

256.
7. Transmisi perinatal hampir tak ditemukan pada
hepatitis
257.
258.
259.

Hepatitis A
8. Transmisi perinatal pada hepatitis B terutama saat

Persalinan

260.

9. Tindakan pada bayi dengan ibu hepatitis B

261.
262.

HBIG dalam 12 jam


10. efek malaria pada janin ?

263.

- Parasit sembunyi dalam plasenta

264.
- mempengaruhi transfer O2 : Abortus spontan , persalinan
kurang bulan,
265.

BBLR, KJDR
266.
11. Ynag bukan merupakan komplikasi pada ibu
hamil
267.
anemia

268.
purpueralis

a.

b. sepsis

269.
selebral

c. malaria

270.

d. KJDR

271.

e. hipoglikemia

272.
12. Saat berbahaya pada ibu hamil pada penyakit
jantung
273.
a.
umur kehamilan
32 34 minggu
274.
b.
partu kala dua

278.

276.
d.
semua benar
277.
e.
semua salah

13. Yang bukan merupakan dampak penyakit jantung

279.
a.
kelainan
organital
280.
b.
pertumbuhan
janin terhambat

284.

275.
c.
pasca persalinan

281.
KJDR

c.

282.
asfiksia

d.

283.
e.
Perdarahan
preterm

14. yang bukan gejala dan tanda penyakit jantung

285.
a.
cepat lelah

286.
ortopneu

b.

287.
c.
bising diastolik
+ bunyi
jantung 2
290.

292.
b.
perdarahan
dalam kehamilan
c. mengetahui
296.

d.

289.
udema

e.

15. yang tidak termasuk indikasi USG

291.
a.
usia kehamilan
terlihat jelas

293.
jenis kelamin

288.
berdebar

294.
d.
perbedaan usia
kehamilan
dengan tinggi
fundus uteri
295.
e.
tersangka
pertumbuhan
janin terhambat

16, yang merupakan tujuan ANC adlah

297.
a.
menyiapkan fisik
dan mental ibu
298.
b.
menyelamatkan
ibu dan anak
dalam
kehamilan,
persalinan, nifas
299.
c.
mengupayakan

ibu sehat fisik


mental pada
masa post
partum
300.
d.
semua jawaban
benar
301.
e.
jawaban A dan
B benar

302.
17. Menurut SDKI 2002/2003 jenis pelayanan ANC
yang paling sedikit diberikan adalah :
303.
a. Info tentang
bahaya kehamilan 28, 7 %

306.
urine 95.3%

d. pemeriksaan

304.
b. timbang berat
badan 89,6%

307.
darah 30.3%

e. pemeriksaan

305.
darah 89.9%

c. ukur tekanan

308.
18. yang tidak termasuk elemen dasar persalinan
adalah :

309.
persalinan

a. waktu

310.
persalinan

b. tempat

311.

c. transportasi

312.

d. dana

313.
keputusan

e. pengambilan

314.
19. pendekatan resiko pada ANC bukan merupakan
strategi yang efektif lagi karena :
315.

a. komplikasi yang tidak dapat diprediksi

316.
komplikasi

b. faktor resiko umumnya bukan penyebab utama

317.
komplikasi

c. banyak ibu hamil dengan resiko rendah mengalami

318.
d. umumnya ibu hamil dengan resiko tinggi bisa
melahirkan dengan selamat
319.

e. semua jawaban benar


320.

20. yang menghasilkan beta HCG :

321.

a. sitotrofoblas

323.

c. hipoblas

322.
las

b.sinsitiotrofob

324.

d. epiblas

325.
326.

21. air ketuban merupakan produk utama dari :

a. selaput ketuban
327.

22. foramen ovale ?

328.
23. putar paksi dalam terjadi saat kepala janin
melalui ?
329.

24. manajemen aktif kala 3 ?

330.

25. penyebab tersering perdarahan post partum?

331.

- atoni

332.
lahir

- perlukaan jalan

333.
- terlepasnya
sebagian plasenta dan uterus
336.

334.
- tertinggalnya
sebagian dari plasenta
335.
-kelainan
pembekuan darah

36. yang mungkin mengalami kehamilan :

337.
a. pubertas
prekok lengkap
338.
b. pubertas
prekok tidak lengkap
341.

339.

c. pubertas tarda

340.
primer

d. pubertas

37. pada manepause

342.
a. gonadotropin
rendah, estrogen rendah

344.
c. godadotropin
tinggi, estrogen rendah

343.
b. gonadotropin
rendah, estrogen tinggi

345.
d. godadotropin
tinggi, estrogen tinggi

346.

38. Salah satu penyebab menopause prekok:

347.
a. unilateral,
salpingoporekromi

349.
bilateral

c. tubektomi

348.
b. total
abdominal histerektomi

350.
d. total
abdominal histerektomi
bisalpingoverektomi

351.
39. pertumbuhan terutama disebabkan oleh
pengaruh :
352.

a. FSH

354.

c. Estrogen

353.

b. LH

355.

d. progesteron

356.
40. indikasi bedah sesar pada kehamilan ganda
adalah dibawah ini kecuali
357.
a. prolaps tali
pusat janin Satu
358.
previa

b. plasenta

359.
c. janin satu
letak sungsang dan janin dua
letak kepala

360.
d. janin
pertama letak kepala dan
janin dua letak sungsang
361.
letak lintang

e. janin pertama

362.
41. Faktor yang tidak berpengaruh terjadinya
kehamilan kembar adalah
363.

a. factor paritas

364.
b. factor
riwayat infertilitas

365.

c. factor umur

366.
keturunan

d. factor

367.

e. factor bangsa
368.
42. Pernyataan mengenai kehamilan kembar dizigot
adalah

369.
telur

a. berasal dari 1

370.
sama

b. rupa dan sifat

371.
c. terdapat pada
1/3 kasus gamelli
374.

373.
sama

e. rupa dan sifat

43. cara mendiagnosis hamil kembar, kecuali

375.
a. terdengar 2
denyut jantung janin pada
tempat agak berjauhan dengan
perbedaan frekuensi sedikitnya
10 denyut per menit
376.
b. teraba 2
bagian besar janin
380.
:

372.
d. mempunyai
2 plasenta, 2 korion, dan
2 amnion

377.
c. uterus lebih
besar dan cepat
pertumbuhannya
378.
d. foto rontgrn
tampak 2 janin
379.
e. ultrasonograf
tanpak 2 janin

44. anemia dalam kehamilan bila kadar hb saat hamil

381.
< 10 gr%

a.

383.
12 gr %

c.

382.
<11 gr%

b.

384.
8 gr %

d.

385.
<7 gr %

e.

386.
45. Bertambahnya darah dalam kehamilan mencapai
puncak waktu umur kehamilan
387.
a.
40-42 minggu

389.
c.
32-36 minggu

388.
b.
38-40 minggu

390.
d.
28-30 minggu

391.
e.
26-28 minggu

392.
46. anemia dalam kehamilan disebabkan oleh
banyak faktor antara lain :
393.

a. malabsorpsi

394.

b. kurang gizi

395.
kronis

c. penyakit

396.
d. kekurangan
zat besi dalan diit
398.

397.

66. Cara mendiagnosis DM dalam kehamilan

399.
a. cukup priksa
glukosa urin
400.
GDP

e. benar semua

401.
c. pemeriksaan
Glukosa darah post
prandial

b. pemeriksaan

402.

d. GTT

403.
67. mana alat kontrasepsi yang paling tepat bagi
seorang ibu DM yang tidak menginginkan kehamilan lagi
404.

a. abstinersia

406.

c. spiral

405.
oral

b. kontrasepsi

407.
d. kontrasepsi
pasca persalinan

408.
68. seorang ibu hamil 20 mg terdapat kadar glukosa
sewaktu >120 mg/dl
409.

a. penderita DM

410.
b. mungkin
diabetes gestasional

411.

c. bukan DM

412.
d. harus
diberikan insulin

413.
Dm Bumil Bila tidak diterapi :

69. Komplikasi

414.
a. Sembuh
sendiri bila sudah partus

416.
IUFD

c. janin dapat

415.

417.
malnutrisi

d. si ibu menjadi

b. janin KMK
418.

70. setiap bumil dg DM diterapi dengan :

419.

a. insulin

420.

b. oral DM

421.
insulin

c. Oral DM dan

422.
d. Mulai oral DM
bila tidak berhasil baru insulin
423.
424.
425.

SOAL UAS OBSGIN 2007

1.

1.

1.

myoma geburt adalah :


a. berasal dari mioma sub serosum
b. berasal dari mioma intramural
c. tumor berkonsistensi kenyal dan warnanya putih
d. konsistensi kenyal dan warnanya hitam
perubahan sekunder mioma uteri,kecuali
a. atropi
c. degenera
b. degenera
si kistik
si hialin
salah satu pengaruh mioma uteri dalam kehamilan :
a. retensio
c. konstipas
uteri
i
b. abortus
e. 9. komplikasi tumor pvarium, kecuali
a. perdarahan
b. sindrom cushing
e. 13. sindrom meigs teriri dari

d. degenera
si putih
d. hipertens
i

c. putaran tangkai
d. sindrome meigs

a. kista
c. hidrotora
dermoid
k
b. asites
d. b,c bener
e. 15. yang termasuk sex cord stromal tumor adalah
a. androbalstoma
c. brenner
b. teratoma
d. koriokarsinoma
e. 21. biopsi terarah yang tidak kasat mata dilakukan dengan
tuntunan :
a. USG
c. Gastroskopi
b. Kolposkopi
d. Histeroskopi
e. 29. petunjuk adanya perbaikan dalam penanganan kasus
dengan syok hemoragik adalah sbb :
a.

tekanan darah sistol


meningkat 90 mm hg
b.
kesehatan membaik

c.

produksi
urine
>100ml/ 4 jam
d.
pernafasan baik dan
saturasi oksigen normal
e. 31. pada syok hemoragik tindakan utama yang harus
dilakukan dengan urutan :

a. stabilisasi, perbaiki sirkulasi dan oksigenasi, penanganan


penyebab
b. perbaiki sirkulasi dan oksigenasi, stabilisas, penanganan
penyebab
c. penanaganan penyebab, stabilisasi dan perbaiki sirkulasi
d. oksigensi, pemasangan infus, stop bleeding
f. 35. mola komplit terjadi karena :

a.
b.
c.
d.

sperma membuahi sel telur yang normal


sel telur oleh 2 sel sperma
sperma membuahi sel telur tak berinti
2 sperma membuahi satu telur berinti
g. 39. PTG yang dominan :
a.

b. Mola
c.
struens
d. 41. koriokarsinoma bermetastase terutama pada organ ;
a. Vagina
c. Hati
b. Otak
d. Paru
e. 43. pada PTG stadium II ( FIGO )
a. MENYEBAR ke paru
c. Kelua uterus masih di
b. Menyebar ke tempat
pelvis
lain
d. Lesi terbatas di uterus
e. 45. kespro adalah kesejahteraan fisik, mental dan sosial yang
utuh dan berhubungan dengan :
a.
b.
c.
d.

Proses kehamilan, persalinan dan nifas pd masa remaja


Proses reproduksi dan komplikasi pada ms remaja
Siklus reproduksi, fungsi, serta prosesnya pada remaja
Fungsi reproduksi dan segala permasalahan pada remaja
f. 47. masalah yang sering menimpa remaja, a.l :

a. KTD
c. Aborsi
b. Narkoba
d. Semua benar
e. 49. resiko hub seksual pada usia muda :
a. Resiko kanker leher rahim
meningkat
b. Resiko aborsi
e. 51. untuk melihat anatomi tuba :

c. Resiko
hamil
kehamilan resti
d. Semua benar

f. Laparoskopi

h. Faloskopi

g. Histerosalpingografi

i. Kolposkopi

dan

j. 53. bila kedua tuba non paten, tindakan berikutnya :


k. Intra uterine inseminasi

m. Senggama terjadwal

l. Picu olukasi

n. In vitro fertilisasi

o. 55. pada dasarnya penanganannya perdarahan uterus disfungsional


pada wanita denga selaput dara intak adalah :
p. Kuret

q. Estrogen

r.

Estrogen dan progesteron

s. Entikoagulansia

t. 57. pemeriksaan ginekologi untuk follow up akseptor AKDR,


pemeriksaan pertama adalah
a.
b.
c.
f.

VT
Inspekulo
Palpasi abdomen
60. hipotesa threshold windows

d. USG
e. Meraba tali AKDR

a. Kadar FSH 10-30 % diatas ambang minimal dalam kurun waktu


tertentu
b. FSH10-30 % diatas kadar ambang minimal
c. FSH 10-30% Lebih rendah dari ambang minimal
d. FSH 10-30% lebih rendah dari ambang minimal dalam eleman
waktu
e. Tak ada pengaruh kadar FSH dan elemen waktu
g. 62. dengan kontrasepsi dimaksudkan.. kecuali :
a. Menunda kehamilan
d. Menghentikan
b. Merencanakan
kehamilan
kelahiran
e. Menyetop kehamilan
c. Menjarangkan
kehamilan
f. 63. pada menopause kadar FSH mencapai
a. >5 MIU/ ML
c. >20
b. >10
d. >30
e.
69.dalam skaa resiko osteoporosis, wanita tak haid 12 24 bulan
mendapat nilai :
a. 1
c. 3
b. 2
d. 4
e. 70. dalam skala resiko osteoporosis pengguna alkohol mendapat
nilai :
f. 1
a. 2
g.

b. 3
c. 4
h.

i. SOAL UTS OBGIN 2007/2008 (persiapan untuk UAS)


1. presentasi bokong dan kedua kaki menggunakan perasat apa?..
2. Kepala sulit lahir namanya apa?menggunakan perasat apa?............
3. masa puerperium adalah satuan waktu yang diperlukan olehseorang
puerpera kembali ke arah sebelum partus. Masa tersebut memerlukan
waktu (minggu):

a. 3
d. 6
b. 4
e. 7
c. 5
4. partograft adalh alat bantu monitoring kala berapa?..........
5.
hiperemis gravidarum penyebab pastinya tidak diketahui tetapi
ada beberapa faktor ynag mempengaruhi adalah?....
6.
7. soal saraf :
8. 40. parase N hypoglosus dextra ditemui gejala berupa :
9.

1. lidah terdorong ke kanan saat digulirkan keluar

10.

2. mulut tertarik ke kanan

11.

3. mulit tertarik ke kiri

12.

4. lidah terdorong kekiri saat dijulurkan keluar


13.
14.
15.
SOAL-SOAL

16.
17.
1. Preeklampsia dikasi obat dulu atau langsung mgSO 4? Ingat DOSIS
& SYARAT PEMBERIAN!!!
18.
Pemberian antihipertensi > 160/110 mmHg dan MAP 128 mmHg
atau Dr Soetomo > 180 mmHg untuk sistolik atau diastolik > 110 mmHg.
(buku merah)
19.
Indikasi untuk terapi antihipertensi (coppage & Sibai, 2004)
1. Antepartum dan intrapartum
20.
-kenaikan tekanan darah persisten sedikitnya 1 jam sistole >
160mmHg/diastole > 110 mmHg/ MAP > 130 mmHg
21.
-Kenaikan TD persisten sedikitnya 30 mnit, sistole >
200mmHg/diastole > 120 mmHg/ MAP > 140 mmHg, trombositopenia,
gagal jantung kongestif
22.
2. Postpartum (persisten sedikitnya 30 menit) >160 mmHg/>
105mmHg/MAP 125 mmHg. (pemakaian obat pada kehamilan dan
menyusui
23.
Dosis nifedipin (relaksan otot)obat jenis C, lini
pertama, calcium channels blocker
a. 10-20 mg per oral, diulangi setelah 30 menit; maksimum 120 mg
dalam 24 jam. (buku merah)
b. 3x10 mg/hari atau 30 mg sehari. Dosis maksimal idem.
24.
*Nifedipin tdak bleh diberikan sublingual karena efek
vasodilatasi sangat cepat.
25.
Lini kedua

26.
Sodium nitropusside:0,25 mkrogram i.v/kgBB/menit, infus
ditingkatkan 0,25 mkrogram i.v/Kg BB selama 5 menit.
27. MgSO4
28.
Hanya diberikan sbg obat anti kejang jadi pas kejang
diberikan (eklampsia) atau diberikan selama persalinan sampai
24 jam pasca persalinan walaupun tdak kejang
29.
Initial dose4 gr MgSO4 IV (40% dalam 10 cc) slama 15
menit
30.
Maintenance doseinfus 6 gr dalam larutan RL/6 jam atau
diberikan 4 atau 5 gr IM. Selanjutnya diberikan 4 gr IM tiap 4-6
jam
31.
Syarat-syaratx:
1. Harus tersedia antidotum MgSO4, bila terjadi intoksikasi yaitu kalsium
glukonas 10%=1 g (10% dalam 10 cc) diberikan IV slama 3 menit
2. Refleks patella (+) kuat
3. Frekuensi pernafasan >16x/menit, tdak ada tanda distres nafas
32.
Magnesium dihentikan bila ada tnda2 intoksikasihilangx
refleks tendon (10mEq/liter ato 12 mg/dl), trhentinya pernafasan
(15 mEq/l ato 18 mg/dl), trhentinya jantung (>30mEq/l ato
>36mg/dl)
33.
Diuretikumfurosemid (klo ada edema paru, payah
jantung kongestif, edema anasarka)
34.
Kortikosteroidkehamilan
24-34
mnggu
(mematangkan
paru,
penurunkan
perdarahan
periventrikuler dan kematian janin)bethametasone 12
mg IM 2X dgn jrak 24 jam
35.
2. Jenis Hipertensi, ONSET HIPERTENSI PADA kehamilan,
a. Hipertensi Kronik
36.
Adalah hipertensi yang timbul sebelum umur kehamilan 20
minggu atau hipertensi yang pertama kali didiagnosis setelah
umur kehamilan 20 minggu dan hipertensi menetap sampai 2
minggu pasca persalinan. (obatnya metildopa500-4000 mg
atau nifedipin)
b. Preelampsia-eklampsia
37.
Preeclampsi adalah hipertensi yang timbul setelah 20
minggu kehamilan di sertai dengan proteimuria.
38.
Eklampsi adalah preeklampsi yang disertai dengan kejang
dan atau koma.
c. Hipertensi kronik dengan superimposed preeclampsia
39.
Hipertensi kronik yang disertai tanda-tanda preeclampsia
atau hipertensi kronik disertai proteinuria.

d. Hipertensi gestasional (disebut juga transient hypertension)


40.
Adalah hipertensi yang timbul pada saat kehamilan tanpa
disertai proetinuria dan hipertensi menghilang setelah 3 bulan
pasca
persalinan atau kehamilan dengan tanda-tanda
preeclampsia tetapi tanpa proteinuria dan TD >140/90mmHg
3. Rawat Inap
41.
Preeklampsia ringan (> 140/90mmHg, proteinuria > 300 mg/24
jam atau > 1+ dipstik) dlakukan rawat inap bila:
a. Bila tdk ada perbaikan:TD, kdar proteinuria slama 2 mnggu
b. Adanya 1 ato lbih gjala preeklampsia berat
c. Preeklampsia berat (> 160/110mmHg, proteinuria > 5gr/24 jam ato
+4)
4. HELLP syndrom
Hemolysis (LDH, AST, dan Billirubin indirect), elevated liver enzym
(ALT, AST, LDH), dan Low Platelets Count (<150.000/ml)
Klasifikasi missisipi
42.
Klas1trombo :< 50.000/ml, LDH > 600 IU/l, AST dan / ALT > 40
IU/l
43.
Klas 2trombo 50.000-100.000/ml, LDH > 600 IU/l, AST dan /
ALT > 40 IU/l
44.
Klas 3trombo 100.000-150.000/ml, LDH > 600 IU/l, AST dan /
ALT > 40 IU/l
45.
46.
47.
48.
49.
50.
51.
52.
53.
54.
55.
56.
57.
58.
59.
60.
61.
62.
63.
64.
65.

66.

TDS 140 mmHg atau TDD 90 mmHg pada dua kali pemeriksaan

67.
68.
69.
70.
71.
Keterlibatan end organ*
72.
73.
74.
75. Ya
Tidak
76.
77.
78.
TDS 150 mmHg
TDS 150 mmHg
79.
atau
atau
80.
TDD 90 mmHg
TDD 90 mmHg
81.
82.
83.
84.
85.
86.
Terapi nonfarmakologi
87. Terapi farmakologi
88.
89.
*gangguan fungsi ginjal
90.
Proteinuria (>300 mg/24jam, >1+ dip, >30 mg/dL)
91.
Clcr <110 ml/mnt, serum kreatinin >0,8 mg/dL
92.
Keterlibatan jantung
93.
Hipertrofi ventrikel kiri
94.
Keterlibatan mata
95.
96.
Retinopathy
97.
98.
99.
100.
5. Obat untuk ibu hamil yang boleh dan ndak, yang mana yang
kontraindikasi mutlak mana yang relative ???
6. Hiperemis Gravidarum
101. Tingkat 1=ringanmual muntah terus menerus menyebabkan
penderita lemah, tdak mau mkan, BB trun, dan nyeri epigastrium, nadi
100x/menit, TD trun, turgor kulit kurang, lidah kering, n mta cekung.
(B1 dan B6, Dramamin, Avopreg, Avomin, Torecan); antasida dan anti
mulas.
102. Tingkat 2=sedangmual dan muntah yg hebat mnyebabkan
KU pnderita lbh prah:lemah, apatis, turgor kulit mulai jelek, lidak kering
dan kotor, nadi kecil dan cepat, suhu bdan naik (dehidrasi), ikterus

ringan, BB turun, mata cekung, tensi turun, hemokonsentrasi, oliguri


dan konstipasi. Dapat pula terjadi asetonuria dan dari nafas keluar bau
aseton. (infus dekstrosa 5% sbanyak 2-3 liter dlam 24 jam)
103. Tingkat 3=berat KU jelek, ksadaran sangat mnurun,
somnolen smpai koma, nadi kecil, halus dan cepat, dehidrasi berat,
suhu badan naik, tensi trun sekali, ikterus. Komplikasi yang bersifat
fatal pada SSP (ensefalopati Wernicke) dengan adanya nistagmus,
diplopia, perubahan mental. (infus dekstrosa 5% sbanyak 2-3 liter dlam
24 jam)
7. AKDRdilepas langsung bisa hamil
a. Mekanisme Kerja AKDR
104. Sampai sekarang mekanisme AKDR belum diketahui secara
pasti. Pendapat yang terbanyak adalah bahwa AKDR dalam kavum uteri
menimbulkan reaksi peradangan endometrium yang disertai dengan
sebukan leukosit yang dapat menghancurkan blastokista atau sperma.
105.
Pada AKDR bioaktif mekanisme kerjanya selain
menimbulkan peradangan seperti pada AKDR biasa, juga oleh
karena ion logam atau bahan lain yang melarut dari AKDR
mempunyai pengaruh terhadap sperma.
b. Keuntungan AKDR
106.
AKDR mempunyai keunggulan terhadap cara kontrasepsi yang
lain karena;
1. umumnya hanya memerlukan satu kali pemasangan
2. Tidak menimbulkan efek sistemik
3. Alat itu ekonomis dan cocok untuk penggunaan secara missal
4. Efektivitas cukup tinggi
5. Reversibel.
c. Kontraindikasi Pemasangan AKDR
107.
Kontraindikasi untuk pemasangan AKDR dapat di bagi jadi dua
golongan yaitu :
kontraindikasi relatif :
o mioma uteri dengan adanya perubahan bentuk uterus
o insufisiensi serviks uteri
o uterus dengan parut pada dindingnya, misalnya postseksio
sesarea, enukleasi mioma, dsb.
o Kelainan jinak serviks uteri seperti erosio porsiones uteri.
Kontraindikasi mutlak :
o Kehamilan
o Adanya infeksi aktif pada traktus genitalis
o Adanya tumor ganas pada traktus genitalis
o Adanya metroragia yang belum disembuhkan
o Pasangan yang tidak lestari
d. Waktu pemasangan AKDR
108.
-Setiap waktu dalam siklus haid, yang dapat dipastikan
klien tdak hamil

109.
-hari pertama sampai ke-7 siklus haid
110.
-segera setelah melahirkan, selama 48 jam pertama atau
setlah 4-6 mnggu pascapersalinan; setelah 6 bulan apabila
mnggunakan MAL;atau ktika mndapat haid
111.
-Setelah menderita abortus (segera atau dalam waktu 7
hari) apabila tdak ada gjala infeksitrimester 1, trimester 2
tunda pemasngan 4-6 minggu kcuali jika tnaga terlatih
112.
-selama 1-5 hari stelah senggama yg tdak dilindungi.
113.
8. Kontrasepsi yang dapat cegah PMSKondom
114.
9. KB YANG mempengaruhi siklus menstruasi ??
Cari kontrasepsi yang dapat mencegah menstruasi ???
Cari pilihan kontrasepsi lain, bila pasien tidak mau pasang
AKDR ??
Kontrasepsi pilihan bila ada kista atau mioma ???
Kontrasepsi terbaik bila ibu menyusui ???
o Bila menyusui telah 4 bulan atau 8 bulan gmana ??
115.
10. Kehamilan ektopik terkait AKDR,,, pelajari lagi!!!
116.
11. Ca serviks dan Vagina
117.
Sama2 pos koital bleeding tapi Ca Vagina biasanya
disertai nyeri saat berhubungan (dispareuni) sedangkan Ca
Serviks tidak.
118.
12. Cari persamaan dan perbedaan gejala masing-masing
penyakit infeksi traktus genital !!! terutama perbedaannya !!!
119.
13. Antibiotik untuk masing-masing penyakit infeksi traktus
genitalis,, ingat dosisnya juga !!
120.
14. Pelajari kadar hCG normal ?
Tinggi, bila kadar hCG berapa? Terjadi pada kondisi apa saja ?
Rendah, bila kadar hCG berapa ? Terjadi pada kondisi apa saja ?
121.
122.
123.
124.
KUMPULAN SOAL BLOK 16 (REPRODUKSI) 2013
RAKSASA JAS PUTIH

1.
2. 2. Persalinan jam berapa? Jm 12
3.
4. no 4. ada pasien inpartu. pada VT di dpatkan molase:sutura merapat.
bagaimanakah gamabaran di partograf?
a. 1
b. 0

c. 2
d. 3

5. G3p2A0. Vt pembukaan lengkap, pnurunan hodge 3, ktuban utuh, his


3x/10 menit selama 40-45 dtik. Apa tndakan yg hrus dlakukn. Jwb:
pechkan ktubn.

6. 6. udah lhir 1 mnt,plasenta blm lhir, TFU 3 jri datas pusat.


a. Oksitosin,
b. regang
pusat,

tali

c. masase,
d. cek janin lain
e. cek saldo anda

7. 7. G1 lahir 1 menit yll, plasenta blm lahir, kontraksi baik, keluar


semburan darah, apa yg menyebabkan hal tersebut ?
a. a. luka jalan lahir
b. b.
lepasnya
plasenta

insersi

c. c. sisa selaput ketuban


d. d. robekan dinding rahim

8. Soal no.8 ( lupa bro, pokoknya tentang apn)


9. Anies Dyaning Astuti Bayi sungsang dimana DJJ terdengar keras jawab :
lebih tinggi dari pusat
10. 10. Seorang ibu dtg kRS karena merasa kenceng2. amn : G4P2A0,
keluar lendir dan darah. Ibu jg mngaku mulas sprti pgn PUP. Pmbkaan
7cm, penipisan 25%. Apa tanda pasti persalinan pd ibu?
a. A. keluar darah&lendir
b. B. Dilatasi serviks
11.
12.

c. Dorongan
meneran
(keinginan BAB)

Bidang hodge antara batas bawah simfisis pubis ischiadika : 2-3


12. Apa penatalaksanaan pada kasus no 10

a. Pecahkan ketuban

b. Dll

13. 13. Jam 9.00, pembukaan 7cm, jam brp perkiraan pembukaan
lengkap?
a. A. Jm 10
b. B. Jm 10.30

c. C. Jm 11
d. D jm 11.30

e. E. Jm 12.

14.
15. 15.Soal sama dengan CBT. Kasus KPD ap yg dberikan
penatalaksanaanya? Ampicilin IV
16. 16. Tensi 170/110, baru periksa pertama kali, diagnosis -->peb
17. 17. Px pnnjng bwt preeklamsi (sy jwbx analisa urin)
18. 18. ibu hamil td 140/90 riwayat hipertensi sblm kehamilan proteinuria
plus 3 apa diagnosanya : superimposed preeklamsia
19. 19. Wanita hamil
mengeluh pusing, TD 140/95, sebelumnya TD
normal. Tatalaksana?
a. A. Diberikan analgetik utk mengobati keluhan pusing
b. B. Antikonvulsan utk mencegah kejang
c. C. Antihipertensi utk menurunkan TD
d. D. Vitamin dan Fe utk menambah vitalitas
20. 20.wanita hamil 2 bln, nyeri perut, disertai darah kecoklatan. Teraba
uterus sbesar tlur bebek, servik nyeri goyang, tes hcg positif. Dx?KET

21.

Alief abni 21. Dosis obat yg digunakan untuk terminasi

22.

jawab : prostaglandin 20 mg

23. 22. wanita 35 tahun dengan keluhan kejang. HPHT 22 SEPtember


2012. TD 200/100 mmHg. Edema pretibial +3. Kejang sudah 2 kali.
Manajemen dikasih apa utk keluhan utama pasien tersebut?
a. a. Nifedipin
24.

b. b. mgso4

23. Tujuan terapi anti hipertensi =>

25.

Klo ai jawab : sistol <160, diastol <110 cari lagi

26.
27.
28. 26. Pasutri 3thn blm pnya anak. Suaminya suaminya sering ftness dan
merokok. Yang diperiksa :
a. Morfologi sperma
b. Jumlah semen
29.

c. Testis kecil
d. d.

27. Gimana syarat sperma untuk analisis sperma?

a. Digunakan sperma yg stelah brhub terakhir 48jam


b. Setelah 72jam tdk berhubungan
c. C. Sebaiknya dikeluarkan dg provokatus
30.
31. 29. Lokasi infeksi paling sering dari skenrio 28 adalah dimana? Vagina,
ovarium, tuba falopi, endometrium, forniks posterior
32.
33. 31. Haid jarang, tumbuh rambut halus di atas hidung, dxnya? Pcos
34. 32. Bagiaan yg terkena pd pcos adalah ovarium
35. 33. Obat untuk pcos a. Klomifen b. Metformin
36. 34. Perempuan 30 tahun persentasi janin belakang kepala. Diameter
kepala janin saat lewat PAP? A suboksipito bregmatikus . Yang lainya
lu pa pilihannya
37. 1.suka menggosok''kan ke pantat wanita frotisme
38.

2. Senang liat benda mati, terangsang fetisme

39.
3. Istri Kelebihan seks, 6-7x/hariPenyakit Nymphomaniac,
Wanita Dengan Gairah Seks Berlebihan dan Tak Pernah Terpuaskan
40.
4. Tdk mau berhubungan krna sering berantem dg
suami gangguan aerosal
41.

42. 37. Diangkot,suka menggosok2an


memegang bokong wanita
a. A.vayourisme
b. B.fetitisme

penis

ke

wanita

dan

sering

c. C.macorisme
d. frotisme

43.
44. 39. Ibu hamil datang dengan bukaan 4, 4 jam kemudian diobervasi
mnjd bukaan 6, bagaimana gambaran partograf ?
a. A. Berada di garis
waspada.
b. B. Berada di sebelah
dikiri waspada

c. C. Disebelah kanan
waspada
d. d. Pada Garis tindakan
e. e. Kanan garis tindakan

45. 40. Jawabn = persentasi kemungkinan tinggi hamil slma 12 bln awal
prnikahan
46. 41. Seorang wanita berdarah setelah pascasenggama. Pemeriksaan yg
segera dpt dilakukan?
a. A.pem.IVA
b. b.pem.pap smear

c. c.biopsi endomet
d. d. Huh capek deh.

47. 42. terapi untuk tumor ovarium yang telah menyebar ke daerah di
sekitarnya ?
a. a.histeroktomi ,
b. b. Debulking ,

c. c. salfingoktomi.
d. d. radioterapi

48. 43. Susu basi candida


49. 44. Obat susu basi = ketokonazol
50. Soal 45, wanta kpthan,kuning,bau gatal, swb vgna, parasit lnjong dgn
flagela, jwbnx tricomonas vaginalis,
51. 46. Terapi trichomonas? Metronidazol
52. 47. Perdarahan pervaginam, oue tertutup, ada fluxus,. Jawaban:
kemungkinan abortus iminens
53. 48.perdarahan pervaginam,tfu 2 jari d atas simfisis,ostium tertutup
(skenario sama dg no 46),ap tatalaksana?lupa pilihanx,ingetx ada
progestin dan tokolitik,kemoterapi,
54. 49. Jatuh kepleset, keluhn perdarahan, kondisi janin baik : Solusio
plasenta
55. 50. efek catropril : janin mati, abortus, bbrl, PDA, preterm
56.
57. 52. Mual, apa sebabx? Hcg
58.
59. 54. pasien memeriksakan kehamilan,usianya 35 thun. usia kehamilan
35 tahun. TD 170/110,edema seluruh tubuh. bgaimana tatalaksana?

a. a. dikasi antiHT?
b. b.
akhiri
segera
kehamilan

c. c. observasi sampe
usia
kehamilan
38
minggu

60. 55. Pd khamiln CO meningkt, karna SV meningkt n HR meningkt.


Manakn perxtaan yg sesuai.
a. CO meningkt pd trmstr 1 krna pningktn SV
b. b.
c. c. HR meningkt seiring brtmbhx usia khamilan.
61. 56. SVR trun 5%
62. G1P0A0H0 usia kehamilan 38 minggu, ketuban pecah sejak 14 jam yll,
TFU 30 menit, DJJ 12 12 12, pembukaan 1 cm, cairan ketuban kering. Apa
yg dilakukan?
a. a. Rujuk krna KPD>14 jam
b. b. Resusitasi, rujuk
c. c. Ampisilin 1 g iv, rujuk
d. d. Ampisilin, resusitasi, rujuk (kalo dirumah sakit baru bisa di
resusitasi)
e. e. resusitasi, ampisilin, oksitosin drip, observasi 4 jam, rujuk
63. soal no.58 ( partus kasep)
64. Apa yg dilakukan dokter puskesmas untuk kasus persalinan kala 1.
jawab : resusitasi intrauterin sambil merujuk
65. 60. (soal kasep) sampai d RS apa yg hrs yg lakukan?
a. A. Lngsung sc cz kpd >
12 jam
b. B. Induksi oksi

c. C. Antibio, obser 4 jm
lg

66. Alat kontrasepsi yang tidak mengganggu


sebelumnya (pasien riwayat mioma uteri) adalah
a. Minipil
b. Progesteron

gangguan

pasien

c. kondom

67. Faradilla Elmi cantique Pasien mengeluh nyeri, riwayat pemasangan


akdr 1 bln yg lalu,apa yg harus dilakuk an?
a. Cabut akdr
b. Foto rontgen unt melihat posisi akdr
c. Ibuprofen 3x250mg, minta datang kmbali jk nyeri memberat
68. 63. Mekanisme kerja klomifen? Agonis pd hipofisis
69.
70. 65. Kasus ank hamil tiba2 pdahal ndk pernah koitus.ibunya
bingung.berdasarkan umur kehamilannya (berdasarkan soal diatasnya)
apa yang dapat dilihat pada umur segitu?
71. 66. Plasenta previa, fluxus positif --> opname

72. 67. Diameter pnggul anteriorposterior lbh bsr dr transversal (ktx anak2
antropoid)
73. 68. apa saran dokter dengan pelvis seperti tiu : susah lahir
pervaginam
74. 69. Wanita nyeri perut dan perdarahan. Terakhir haid 8 minggu yg lalu.
Tes kehamilan positif, pd pemeriksaan didapatkan ada infeksi peritoneum
cairan bebas di kavum abdomen. Diagnosis?
a. A.
Kehamilan
terganggu

ektopik

b. B. Mola hidatidosa

75. 70.wanita hamil dgn perdarahan, usia hamil lupa, bayi letak
sungsang,his negtf. Dx?Bayi letak sungsang
76. 71. Lupa
77. 72. ibu sedih berat badan bayinya saat lahir 2250 g, tidak seperti 2
anaknya sebelumnya. Bayi tampak normal. Saat hamil ibu malas minum
susu dan istirahat. Diagnosis?
a. a. Preterm
b. b. Posterm
c. c.
berat
rendah

d. d. berat badan bayi ringan


e. e. berat badan bayi kecil
badan

bayi

78. 73. bblr, aterm =>ncb-kmk


79.
80.
81. 76. Letak kepala di fundus. Bokong di bawah. Diagnosis?
82.
83.
84. 79. untuk tes selanjutnya bahwa tidak hamil adalah?
85.
86. 81. Yg nyebabpin cin III? HPV no 16
87. 82. Mek. Kerja IUD adalah menyebabkan peradangan steril pada
daerah uterus
88. 83. Sutura frontalis, ubun-ubun besar, pangkal hidung, lingkar orbit.
Termasuk persentasi apa? Jwbn dahi
89. 84. Seorang laki2 terjadi ganguan paraphilia. Senang melakukan seks
dengan barang2 tertentu seperti celana dalam wanita, . Kelainan pada
laki2 tersebut disebut fetisme
90. 85.wanita 25th, ku: perdarahan pervaginam+mules, hamil 2 bln, vital
sign normal, tfu tdk teraba, pembukaan porsio +, jaringan +, besar
uterus 6-8 mgg, dx?
a. A.iminens
b. b.inkom
plit
91.

c. c.komplit
d. d.abortus
e. e.ovum

92. 87. Penyebab ereksi...cavernosus atau pudendus


93.
94. 89. Ditemukan clue cell, diagnosis apa ? Bakterial vaginosis
95. 90. Knpa fetus bisa makrosomia,. Sma kyk kakak tingkat soalnya
pilihannya yg bner E
96. 91. (what the kampret scenario dari si pengumpul...???) sepasang
suami istri datang mengeluh karena skenariony puaaanjaaang bangett.
Terus mereka bosen soalnya kepanjangan. Minta diperiksa tp dokternya
jg males soalx skenariox panjang benget gitu deh. Akhirnya pasangan
tersebut memutuskan untuk mengubah skenario jd satu kata aja yaitu
dan adalah CLUE-CELL. Akhirnya dokternya dpt mendiagnosa!
a. A.trikomonas vaginalis
b. b.bakterial vaginosis
97.

c. c.panjangitis

92. terapi untuk vaginitis atropikan ?


a. a.
progesteron
transdermal
b. b.
transdermal

estrogen

c. c. estrogen intravagina
d. d.
progesteron
intravagina

98.
99. 94. Blind ovum mau di abortus provokatus = prostaglandin
100. Soal 95. Obat trikomonas vagnalis, jwbnx metronidazol
101. 96. W 32 th dtg tgl 11 april 2011. Hpht 4-8-2010. Tfu 29. G2P1. Anak
prtama 3 th. Td 140/90. Usia kehamilan? A.35 minggu
102. 97. Tujuan ANC...??
103. 98.pasutri umur 27 dan 32 thun ke puskesmas,konsultasi kb,istri bru
melahirkan 3 bulan dan mw menjarakkan kehamilan slma 3
th.kontrasepsi yg sesuai:
a. A.kalender
b. B.kondom

c. C.akdr
d. D.implant

104.
105.
106.
107.
108.
1.

99.kb cegah pms : kondom


100. bishop score8
Bentuk estrogen dijaringan adiposa? Estron
Tafsiran berat janin melalui rumus perhitungan TFU?

36. Istri mengalami penurunan keinginan hub. Seksual akibat


bertengkar dengan suami dan memikirkan masalah trsebut
terus. Gangguan hub sek yg d alami istri?
2. 86. Histerektomi. Ligamentum sebelah anterior uterus apakah
yg ikt terangkat?
109.
110.
SOAL CBT
112.
1. Robekan jalan lahir media labia minora frenulum
111.

2. Riwayat HT, terus berdarah, hamil, jatuh di kamar mandi (APB ec


solusio/trauma)
3. jumlah sperma dan motilitas kurang oligozoospermia,
asthenozoospermia
4. anamnesis untuk risiko anemia pada bumil sekarang, anaknya 3 tahun
baik2 saja----- Tanya diet ibunya
5. pasien datang ke ginekologis, untuk dilakukan px rutin, ada proliferasi
endometrium, anemia, px yang lain normal DUB
6. nyeri pas haid, haid ireguler, laparoskopi ditemukan colored brown
blablablaaa endometriosis
7. jerawat, hirsutisme, usg string pearl, pembesaran adnexa bilateral
diagnosis?
8. jerawat, hirsutisme, usg string pearl, pembesaran adnexa bilateral
patofisiologi pembesaran adnexa small cyst w/ benign, small cyts w/
maligna,
9. ditemukan discharge seperti susu basi, ditemukan hifa candidiasis
vaginalisssss
10.

discharge ada protozoa berflagela metronidazole

11.
mengeluh keluar cairan 3 jam yang lalu, belum ada mules dan
cairan disertai darah KPD

12.
mengeluh keluar cairan 3 jam yang lalu, belum ada mules dan
cairan disertai darah beri ampisilin, injeksi antibiotik
13.
asi, anak DHF, jadi ibu ngk bisa ngasih susu (again) langsung
pk KB saat itu juga brooo
14.

ibu HT 150/sekian mmHg kasi labetolol 10 mg IV

15.
siklus 20 hari, mau sekkolah ke luar negeri, mau pk kb kalender,
sbg dokter gmn pk kb lain, jangan senggama hari 8-19, pk
kontrasepsi hari 8-19
16.
perdarahan UK 23 kehamilan, perut tegang, perdarahan
solusio plasenta
17.
HT sejak kehamilan anak pertama, hamil sekarang yang ketiga
itu hipertensi juga dan proteinuria +3 superimposed preeklamsia
18.
Ibu tidak punya anak 8 bulan, karena bapaknya kerja keluar kota,
senggama 2 minggu sekali senggama 3x/minggu
19.

Perdarahan banyak berkumpal menometroragia

20.

HPHT 1 mei 2012 HPL 8 Februari 2013

21.
Gangguan haid,wanita 20 tahun, tidak ada pertumbuhan
payudara gangguan aksis hipotalamus-hipofisis, gangguan pada
ovarium
22.

Ballotement (-), DJJ (-), tes hamil positif mola hidatidosa

23.
selama kehamilan ada peningkatan CO, HR dsb..apa pernyataan
yg benar ? HR terus meningkat smpe trimester III, CO menurun pd
trimester 3,
24.
seorang ibu 35 th , mengalami keguguran 4 kali berturut2 ,
pemeriksaan yang perlu dilakukan ?
HbSAg , HPV , Tiroksin estrogen progesteron , kromatin seks , autoimun
113.

Anda mungkin juga menyukai